Download Respiratory Sciences

Document related concepts

List of medical mnemonics wikipedia , lookup

Transcript
Respiratory Sciences
Question 1
Deficiencies in medical knowledge are rarely the cause of errors in clinical reasoning but
errors in diagnostic or management reasoning are common. Which of the following best
defines "confirmation bias"?
A. The tendency to do something (or seen to be doing something) even if intended actions
are not supported by robust evidence and may do harm.
B. The tendency to over-treat or under-treat in the hope that certain events will (or will
not) happen, rather than acknowledge what is statistically most likely to happen.
C. The tendency to opt for candidate diagnoses based on how the problem is perceived or
framed according to past history, clinical setting, previous diagnostic labels, and other
contextual factors.
D. Selective seeking out of information that seems to confirm favoured diagnosis and
ignoring data that are inconsistent with the diagnosis.
E. The tendency to accept a diagnosis because of ease in recalling a past similar case
rather than on the basis of prevalence or probability.
Question 2
Which of the following is TRUE with regards to obstructive sleep apnoea (OSA)?
A. Mandibular splints are 100% effective if OSA is the result of an orofacial abnormality.
B. Tracheostomy is not effective as a sole procedure when there is an underlying specific
abnormality.
C. Continuous positive airway pressure (CPAP) is the most efficacious treatment, but
compliance and correct use are issues.
D. OSA is characterised by repeated lower airways obstruction during sleep.
E. OSA is always symptomatic, and risk of developing increases with age, smoking and
central obesity.
Question 3
Which of the following is not a diagnostic criteria for obstructive sleep apnoea (OSA)?
A.
B.
C.
D.
E.
Excessive daytime sleepiness unexplained by other factors
Choking during sleep
Impaired concentration
Unrefreshing sleep
2 or more apnoeas per hour of sleep
Question 4
Which of the following is not commonly acknowledged to cause drug-induced (iatrogenic)
interstitial lung disease (ILD)?
A. Bleomycin
B. Methotrexate
C. Amiodarone
D. Dapsone
E. Nitrofurantoin
Question 5
Which of the following is not a feature of interstitial lung disease (ILD)?
A.
B.
C.
D.
E.
Inspiratory wheeze
Early productive cough
Digital clubbing
Inspiratory crackles
All of the above are features of ILD
Question 6
When spirometry is performed, some important measurements which can be reported
include forced expiratory volume in one second (FEV1), forced vital capacity (FVC) and the
FEV1/FVC ratio.Which of the following spirometry results would be suggestive of
interstitial lung disease (ILD)?
A.
B.
C.
D.
E.
Increased FEV1, decreased FVC and an increased FEV1/FVC ratio.
Decreased FEV1, decreased FVC and an increased FEV1/FVC ratio.
Decreased FEV1, decreased FVC and a decreased FEV1/FVC ratio.
Increased FEV1, increased FVC and an increased FEV1/FVC ratio.
Increased FEV1, decreased FVC and an increased FEV1/FVC ratio.
Question 7
Which of the following is least useful for specific diagnosis of interstitial lung disease (ILD)?
A.
B.
C.
D.
E.
High-resolution computed tomography (CT) scan
Spirometry
Transbranchial biopsy and bronchoalveolar lavage (BAL)
Autoimmune screen
Histopathology
Question 8
Which two drug classes are most commonly used to treat long-term, stable chronic
obstructive pulmonary disease (COPD)?
A.
B.
C.
D.
E.
β-adrenoceptor agonists, anticholinergics.
β-adrenoceptor agonists, corticosteroids.
β-adrenoceptor agonists, theophylline.
Anticholinergics, corticosteroids.
Anticholinergics, theophylline.
Question 9
Which of the following pathological changes in the lung is not generally characteristic of
chronic obstructive pulmonary disease (COPD)?
A.
B.
C.
D.
E.
Mucous hyper-secretion.
Pulmonary hypertension.
Reduced lung compliance.
Fibrosis of the small airways.
Abnormal gas exchange in the alveoli.
Question 10
Which of the following is an appropriate treatment for breathlessness in patients with
chronic obstructive pulmonary disease (COPD)?
A.
B.
C.
D.
E.
Bronchodilator use.
Participation in an exercise program.
Supplemental oxygen therapy.
Interventions to improve the patient's mood and self-efficacy.
All of the above.
Question 11
Which of the following statements is incorrect, regarding cystic fibrosis (CF)?
A. CF is Australia’s most common genetically inherited disease.
B. The average life expectancy at birth for people with CF is currently 37 years.
C. CF is not a treatable condition, however with good management, disease progression
can be slowed.
D. Medications such as salmeterol, terbutaline and Inhaled corticosteroids can be used to
manage CF.
E. CF is an autosomal dominant disorder, and affected persons are usually identified at
birth.
Question 12
Which of the following is evidence against the traditional hypothesis about the
pathogenesis of idiopathic pulmonary fibrosis (IPF)?
A. There are usually signs of granulomas and increased amounts of lymphocytes.
B. Over 35% of IPF patients have signs of caseating necrotic granulomas.
C. Histologically, there are very few signs of inflammation and anti-inflammatory drugs
don’t successfully treat the illness.
D. Prognosis of IPF is too poor and at the moment, there are no available treatments other
than those being tested in clinical trials.
E. Some IPF is caused by hypersensitivity and as a result, fibroblast and myoblasts
deposits type-1 collagen into extra-cellular matrix.
Question 13
On a coronal CT of the head and neck displaying the cavernous sinus, within this sinus,
which of the following structures are located immediately inferiorly to the oculomotor
nerve (CN III)?
A. The internal carotid artery
B.
C.
D.
E.
The trochlear nerve (CN IV)
Meckel’s Cave
The opthalmic branch of the trigeminal nerve (CN V)
The optic branch of the trigeminal nerve (CN V)
Question 14
Which muscle is not innervated by the hypoglossal nerve (CN XII)?
A.
B.
C.
D.
E.
Palatoglossus
Hypoglossus
Genioglossus
Styloglossus
The intrinsic muscles of the tongue
Question 15
Which of the following signs is consistent with a lesion of the hypoglossal nerve (CNXII)?
A.
B.
C.
D.
E.
Absent gag reflex
Sense of taste is diminished
Patient cannot shrug his shoulders
Patient pokes out his tongue and it deviates to the unaffected side
Patient pokes out his tongue and it deviates to the affected side
Question 16
At the end of a normal (non-effortful) inspiration, what are the most likely pressures to be
recorded in the intrapleural space and alveolar regions (assuming mouth pressure is 0
cmH2O)?
A.
B.
C.
D.
E.
Intrapleural pressure +5 cmH2O, alveolar -1 cmH2O
Intrapleural pressure 0 cmH2O, alveolar 0 cmH2O
Intrapleural pressure -8 cmH2O, alveolar 0 cmH2O
Intrapleural pressure -5 cmH2O, alveolar -1 cmH2O
Intrapleural pressure -8 cmH2O, alveolar -1 cmH2O
Question 17
What is the first thing you should check and comment on when reading a chest X-ray
(CXR)?
A. Check the CXR was taken at full inspiration by counting and ensuring the 6th anterior
rib is visible.
B. Check that the trachea is mid-line and not deviated.
C. Look for flattening in the diaphragm and the costo-phrenic angles.
D. Note the patients name and date of birth.
E. Look for lytic lesions in the bones.
Question 18
Which of the following suggests a sufficient level of inspiration when viewing a chest x-ray
(CXR)?
A.
B.
C.
D.
E.
You can view the 8th anterior rib at the mid-clavicular line.
The diaphragm will be contracted and flattened.
The diaphragm will be relaxed and even on both sides.
You can view the 6th anterior rib at the mid-clavicular line.
You will be able to see the air inside the lungs.
Question 19
Which of the following is NOT generally a cause of low arterial oxygen levels
(hypoxaemia)?
A.
B.
C.
D.
E.
Hyperventilation.
Ventilation/perfusion mismatch.
Diffusion defect across alveolar-endothelial wall.
Right to left shunt.
Breathing at altitude.
Question 20
Which of the following is the least likely cause of type 2 respiratory failure (hypoxia with
hypercapnia)?
A.
B.
C.
D.
E.
Obstructive lung diseases.
Opioid overdose.
Guillain-Barré syndrome.
Kyphoscoliosis.
Low ambient oxygen levels at altitude.
Question 21
Varenicline is a smoking cessation therapy that functions via which of the following
pharmacological mechanisms? (mAChR = Muscarinic Acetylcholine Recptor, nAChR =
Nicotinic Acetylcholine Receptor)
A.
B.
C.
D.
E.
mAChR antagonist
nAChR agonist
nAChR antagonist
nAChR partial agonist
mAChR partial agonist
Question 22
A patient presents with difficulty protracting their jaw and deviation of their jaw to the
right side of their face. This is likely associated with a problem with which muscle(s)?
A.
B.
C.
D.
The right temporalis muscle.
The left temporalis muscle.
The right medial and lateral pterygoid muscle.
The left medial and lateral pterygoid muscles.
E. The right buccinator muscle.
Question 23
Which of the following is false, regarding emerging infectious diseases (EIDs)?
A.
B.
C.
D.
E.
EIDs are increasing in incidence.
30-40% originate from wildlife.
Current incidence rates are sporadically distributed globally.
Current incidence rates of non-zoonotic EIDs show an increasing trend.
The peak number of cases of a zoonotic EID in wildlife will precede that of humans.
Question 24
What are three effects of hypoxia-inducible factors (HIFs)?
A. They increase levels of growth factors, erythrocyte generation and decrease the rate of
lactate production.
B. They increase rates of hydrogen sulfide production, increase levels of vascular
endothelial growth factor (VEGF) and increase rates of angiogenesis.
C. They decreased rates of hydrogen sulfide production, decrease levels of pyruvate
dehydrogenase kinases and decrease the rate of angiogenesis.
D. They cause increased cell survival and expansion, increased levels of vascular
endothelial growth factor (VEGF) and decreased rates of lactate production
E. They cause increased rates of lactate production, increased numbers of erythrocytes in
blood and increased oxygen delivery to tissues.
Question 25
What percentage of men with cystic fibrosis are also found to have CBAVD (congenital
bilateral absence of the vas deferens)?
A.
B.
C.
D.
E.
50-74%
75-89%
90-94%
95-99%
100%
Question 26
In examining a patient's arterial blood gas results (obtained while breathing room air) you
observe the following values: pO2 = 55 mmHg, pCO2 = 37 mmHg. What is the
corresponding A-a DO2 (Alveolar - arterial gradient)?
A.
B.
C.
D.
E.
44 mmHg
49 mmHg
58 mmHg
65 mmHg
69 mmHg
Question 27
Which of the following most correctly describes the changes in electrical brain activity as
you go from wake to sleep, as shown on EEG (electroencephalography)?
A.
B.
C.
D.
E.
Slowing and increased synchronisation.
Slowing and increased α activity.
Faster and increased α activity.
The brain patterns slow.
None of the above.
Question 28
The "fight or flight" response, which encompasses many acute physiological responses to
stress, is triggered by the arousal of which nervous system (specifically)?
A.
B.
C.
D.
E.
The autonomic nervous system.
The sympathetic nervous system.
The peripheral nervous system.
The central nervous system.
The para-sympathetic nervous system.
Question 29
The currently accepted model of stress is the transactional model. This model regards...?
A.
B.
C.
D.
E.
Stress as a process of transaction between environment and person
Stress as external to the person
Stress as a focus on the behavioural response of the person
Stress as concerned with dealing with symptoms, not response
Stress as a "bad" thing
Question 30
Most patients with emphysema usually have more damaged alveoli in the upper zones of
the lungs, a reason for this is that...
A. The upper zones of the lungs undergo more mechanical stress than in the lower zones.
B. There are more pollutants trapped in the upper zones of the lungs.
C. The upper zones of the lungs have less blood flow than the lower zones, therefore
necrosis is more likely to occur.
D. There is more surfactant coating on the alveoli in the upper zones of the lungs than in
the lower zones.
E. There is more gaseous exchange in the upper zones of the lungs than in the lower zones.
Question 31
According to the incidence of age-standardised lung cancer for men and women over the
last 30 years, how does the incidence rate compare in 2010 to the incidence rate in 1980?
A.
B.
C.
D.
It was lower for both men and women.
It was higher in men and lower in women.
It was lower in men and showed no change in women.
It was lower in men and higher in women.
E. It showed no change in men and was higher in women.
Question 32
What is not a risk factor for the development of OSA?
A.
B.
C.
D.
E.
Male
Central obesity
Genetics
Hypothyroidism
Narcolepsy
Question 33
Which of the following is a feature of ILD?
A.
B.
C.
D.
E.
Increased lung compliance.
Low forced expiratory volume to forced vital capacity ratio (FEV1/FVC ratio)
Increased diffusing capacity.
Reduced lung volume.
Floppy lungs.
Question 34
Which of the following is not commonly associated with COPD?
A.
B.
C.
D.
E.
Dyspnoea on exertion
Wheeze
Stridor
Anorexia
Ankle odemea
Question 35
What is the principle mechanism of hypoxemia in patients with COPD?
A.
B.
C.
D.
E.
Hypoventilation
Diffusion impairment
VQ inequality
Shunt
Abnormal Haemoglobin
Question 36
Posttraumatic stress disorder (PTSD) has been described as "the complex somatic,
cognitive, affective, and behavioral effects of psychological trauma" Which of the following
does NOT characterise PTSD?
A.
B.
C.
D.
E.
intrusive thoughts
Nightmares and flashbacks
Alcohol abuse
Avoidance of reminders of trauma
Hypervigilance
Question 37
First-responders / rescuers to disasters are prone to experiencing PTSD. Which of the
following is currently NOT a driver for experiencing PTSD?
A.
B.
C.
D.
E.
Exposure to mutilated bodies / injured people
Feeling of helplessness
Shame regarding post-traumatic symptoms
Survivor Guilt
Lack of Support from Authorities
Question 38
Regarding chronic obstructive pulmonary disease (COPD), which of the following is FALSE?
A.
B.
C.
D.
E.
β2 antagonists can be used to reduce the symptoms of COPD with moderate effect.
The disease process is different from asthma, with different cells involved.
Lung function can never be returned to normal, even with medication.
Onset is usually after 45 years of age.
In advanced stages of the disease, Inhaled corticosteroids have no effect on the decline
in lung function but they can reduce exacerbations of the disease.
Question 39
Which of the following statements regarding the circadian system and disruption to this
system is FALSE?
A. The health effects of shift work include heart attacks, strokes, obesity and reduced
immune function.
B. The health effects of shift work include heart attacks, strokes, weight loss and reduced
immune function.
C. Risk factors for drowsy driving crashes include young adult males, late night/early
morning periods and night shift workers.
D. Body temperature is lower during the night and higher during the day.
E. Higher melatonin levels during puberty can cause circadian disruption.
Question 40
Which of the following statements about the organisation of the pulmonary hilum is
FALSE?
A.
B.
C.
D.
E.
The pleural sleeve drapes inferiorly to create the pulmonary ligament.
The pulmonary arteries are always posterior to the bronchi.
The groove for the aortic arch lies behind the pulmonary hilum of the left lung.
In the left and right hila, the veins are always the most anterior and inferior holes.
The parietal and visceral pleura are continuous at the hilum.
Question 41
Which of the following statements about vaccines is FALSE?
A. Live vaccines may retain some pathogenicity and revert to virulence.
B.
C.
D.
E.
Certain vaccines have been shown to increase the risk of developing autism.
Live vaccines can lose their potency if they are exposed to heat.
Maternal antibodies can interfere with the development of active immunity is a child.
Live measles vaccine can be given to immunocompromised persons.
Question 42
Which of the following statements regarding the use of a large volume spacer when taking
asthma inhaler medications is FALSE?
A. It reduces local side effects by minimising deposition of inhaled corticosteroids in the
oropharynx.
B. A spacer allows for more medication to be delivered to the lungs than an inhaler alone.
C. It is easier to co-ordinate timing of activating the inhaler and inhalation.
D. A spacer is not effective for bronchodilator delivery during emergency situations.
E. None of the above.
Question 43
Which of the following statements is FALSE?
A. Meconium ileus is associated with cystic fibrosis.
B. G551D is a mutation which causes reduced movement of chloride ions into the airway
lumen.
C. Pulmozyme aids in the clearance of mucous from the airways in cystic fibrosis patients.
D. Life expectancy for cystic fibrosis patients has increased over the last 30 years.
E. Cystic fibrosis patients typically have a normal FEV1.
Question 44
Which of the following is TRUE?
A. Respiratory failure occurs when arterial oxygen pressure drops below 75 mmHg.
B. At 80% arterial oxygen saturation, there is a risk of rapidly worsening respiratory
failure.
C. Arterial carbon dioxide is always increased in respiratory failure.
D. Respiratory failure will result in an increased drive to breathe.
E. Supplied oxygen will always improve respiratory failure to some extent.
Question 45
In regards to management of sleep loss and circadian disruption, which of the following is
TRUE?
A. Prophylactic or preventative naps of a minimum of 30 minutes are recommended.
B. Modafinil is a benzodiazepine drug that should be avoided.
C. Administering exogenous melatonin in the morning will cause phase advancement in
the sleep-wake cycle.
D. Melatonin is contraindicated in adolescents, pregnant women and people with
cardiovascular disease.
E. Lorentin is a stimulant drug that has side effects of nausea and diarrhoea.
Question 46
Which of the following about the lung is TRUE?
A.
B.
C.
D.
E.
Collagen fibres in the lung gives the lung its elasticity.
The number of type III collagen fibres in the lung tissue decreases with age.
A person's lung volume is determined by their race and height.
Alveolar size is determined by a person's race and height.
Lung elastic recoil is affected by a person's height and race.
Question 47
Which of the following statements regarding the transport of oxygen in the respiratory
system is TRUE?
A.
B.
C.
D.
E.
Oxygen moves the major airways by convection.
Oxygen is at its highest partial pressure in the body when it is in the arteries.
Oxygen moves across the alveolar-capillary membrane via convection.
Oxygen is lost to anatomical dead space as it passes through the pulmonary vasculature.
Oxygen undergoes diffusion across the terminal airways.
Question 48
Which of the following statements about the diaphragm is TRUE?
A. The oesophagus pierces the muscular diaphragm posterior to the central tendon at the
level of thoracic vertebra 8.
B. On expiration, the right dome is at the level of the 5th intercostal space.
C. The left and right crus join to form the median arcuate ligament.
D. The lateral arcuate ligaments span over the psoas major muscles.
E. The inferior arterial blood supply comes from the thoracic aorta.
Question 49
Which of the following statements regarding forced expiration is TRUE?
A. A person's height does not affect their peak expiratory flow rate.
B. Peak expiratory flow rate is limited by a person’s ‘flow limiting segment’, which is
where the pressure inside the airway lumen has becomes the same as the pleural
pressure. After this point, the airway lumen narrows.
C. Peak expiratory flow rate is limited by a person’s ‘flow limiting segment’, which is
where the pressure inside the airway lumen has becomes the same as the pleural
pressure. After this point, the airway lumen widens.
D. A person's peak expiratory flow rate increases with their age.
E. The normal FEV1/FVC ratio (forced expiratory volume in 1 second to forced vital
capacity) is 98%.
Question 50
Which of the following statements about the cranial nerves is TRUE?
A. The corneal reflex tests the integrity of pontine pathways.
B. The motor fibres of the accessory nerve (CN XI) only innervate the trapezius muscle.
C. Bell's palsy is only ever caused by an extra-cranial lesion.
D. The olfactory (CN I) and optic nerves (CN II) are the only cranial nerves without motor
fibres.
E. The maxillary branch of the trigeminal nerve (CN V) passes through the superior orbital
fissure.
Question 51
Patients with classic cystic fibrosis have abnormally functioning ion channels on the apical
surface of certain cells. Which of the following statements is TRUE?
A.
B.
C.
D.
E.
There is increased resorption of sodium in the sweat duct.
There is decreased resorption of chloride in the sweat duct.
There is increased secretion of sodium in the respiratory epithelium.
There is decreased resorption of chloride in the respiratory epithelium.
There is decreased water resorption in the respiratory epithelium.
Question 52
Which of the following factors has been shown to have the greatest effect on incidence of
CF?
A.
B.
C.
D.
E.
Maternal smoking during pregnancy
Sex
Ethnicity
Premature birth
Malnutrition during pregnancy
Question 53
Which of the following is a not a goal of wholistic physiotherapy management of patients
with CF?
A.
B.
C.
D.
E.
Maximise and maintain aerobic fitness, functional strength and muscle bulk
Efficient sputum clearance
Prevent and treat stress incontinence
Advise patients on exercises to lose weight to assist sputum clearance
Promote and maintain good posture
Question 54
What is the most common mode of inheritance for CF?
A.
B.
C.
D.
E.
Autosomal dominant
Autosomal recessive
X-linked dominant
X-linked recessive
Complex inheritance
Question 55
In describing lymph lode levels of the neck, what is the approximate location of neck level
III?
A. In the anterior triangle from the skull base to the inferior border of the hyoid bone.
B. In the anterior triangle from the inferior border of the hyoid bone to the inferior border
of the cricoid cartilage.
C. In the anterior triangle from the inferior border of the hyoid bone to the clavicle.
D. In the anterior triangle under the thyroid gland.
E. In the posterior triangle.
Question 56
A patient's arterial blood gas results are shown below. Which option correctly describes
these results?pH = 7.35Partial pressure of oxygen: 70 mmHgPartial pressure of carbon
dioxide: 60 mmHgBase excess = +12 mmol/L
A.
B.
C.
D.
E.
Uncompensated respiratory alkalosis.
Compensated metabolic acidosis.
Uncompensated respiratory acidosis.
A normal alveolar-arterial (A-a) gradient.
None of the above.
Question 57
Which of the options correctly describes the arterial blood gas results listed below?pH:
7.54Partial pressure of oxygen: 126 mmHgPartial pressure of carbon dioxide: 17
mmHgBase excess: -3 mmol/L
A.
B.
C.
D.
E.
There is a pathological A-a gradient
Overcompensated respiratory acidosis
Uncompensated metabolic alkalosis
Compensated respiratory alkalosis
Uncompensated respiratory alkalosis
Question 58
What is the most likely interpretation of the following arterial blood gas (ABG)? PaO2 = 119
mmHgPaCO2 = 21 mmHgpH = 7.55Base Excess = -4 mmol/L
A.
B.
C.
D.
E.
Respiratory alkalosis, partially compensated
Metabolic alkalosis, partially compensated
Respiratory alkalosis, uncompensated
Metabolic alkalosis, uncompensated
Respiratory alkalosis, fully compensated
Question 59
Which of the options correctly describes the arterial blood gas results listed below?pH:
7.38Partial pressure of oxygen: 98 mmHgPartial pressure of carbon dioxide: 38 mmHgBase
excess: -8 mmol/L
A. Uncompensated respiratory alkalosis.
B.
C.
D.
E.
Compensated metabolic acidosis.
Compensated respiratory acidosis.
A large A-a gradient.
Uncompensated metabolic alkalosis.
Question 60
In a forced expiration, what are the most likely pressures measured at each of the following
structures or spaces: A) chest wallB) intrapleural spaceC) pulmonary elastic recoilD)
alveolusNote: all measurements in cmH2O
A.
B.
C.
D.
E.
A) +5 B) -5 C) +8 D) +3
A) 0 B) 0 C) +8 D) +3
A) -7 B) +7 C) +9 D) +10
A) -5 B) +5 C) +8 D) +13
A) +5 B) +5 C) +8 D) +13
Question 61
Which statement is INCORRECT?
A. Intense emotion can trigger an asthmatic attack.
B. Epidemiologically, prevalence of atopic dermatitis tends to peak in the late teenage
years.
C. Chronic nasal blockage during childhood can bring about characteristics such as
breathing through the mouth and distortion of growth in the central part of face (e.g. a
wide nasal bridge) referred to as "allergic facies".
D. Dust mites are an example of a perennial allergen.
E. Allergic sensitisation to peanuts can come about before a child has ever consumed
peanuts.
Question 62
A consequence of chronic diurnal sleep restriction include all of the following, EXCEPT
A.
B.
C.
D.
E.
increased incidence of gastrointestinal distrubances
Increased driving related accidents
increased reaction/response time
increased immune function
increased risk of miscarriage
Question 63
Asthma is an example of a
A.
B.
C.
D.
E.
Type 1 hypersensitivity
Type 2 hypersensitivity
Type 3 hypersensitivity
Type 4 hypersensitivity
Type 5 hypersensitivity
Question 64
Which of the following is a modifiable risk factor for Obstructive Sleep Apnoea?
A.
B.
C.
D.
E.
Age
Male gender
Smoking
Endocrine disorders
Allergic disease
Question 65
Cessation of airflow at the mouth needs to occur for how long to be classed as an apnoea?
A.
B.
C.
D.
E.
2 or more seconds
3 or more seconds
5 or more seconds
10 or more seconds
15 or more seconds
Question 66
Which of the following criteria MUST be fulfilled for a diagnosis of obstructive sleep
apnoea?
A.
B.
C.
D.
E.
Recurrent nocturnal awakening
Choking during sleep
Excessive daytime sleepiness unexplained by other factors
Demonstrated respiratory disturbance index >= 5/hour
Sound recording and analysis of snoring
Question 67
Which of the following is the main muscle is implicated in obstructive sleep apnoea?
A.
B.
C.
D.
E.
Tensor veli palatini
Genioglossus
Geniohyoid
Thyrohyoid
Levator veli palatini
Question 68
Which of the following is NOT a commonly cited risk factor for obstructive sleep apnoea?
A.
B.
C.
D.
E.
Alcohol consumption
Hypothyroidism
Male Gender
Obesity
Growth hormone deficiency
Question 69
Which of the following signs and symptoms is NOT consistent with a diagnosis of sleep
apnoea?
A.
B.
C.
D.
Excessive daytime sleepiness unexplained by other factors.
Choking during sleep.
Recurrent nocturnal wakening.
Overnight monitoring demonstrates 5 or more apnoeas plus hypopnoeas per hour of
sleep.
E. Difficulty initiating sleep.
Question 70
Which of the following groups of symptoms are all risk factors for sleep apnoea?
A.
B.
C.
D.
E.
Obesity, male gender and hypertension.
Alcohol use, anatomically small airway and immune deficiency.
Obesity, a history of hypothyroidism and a history of diabetes.
Obesity, alcohol use and an anatomically small airway.
Hypotension, alcohol use and a history of smoking.
Question 71
Which of the following is the most appropriate definition of dyspnoea?
A.
B.
C.
D.
E.
A subjective awareness of discomfort related to the act of breathing.
A subjective sensation of pain related to the act of breathing.
A breathing rate greater than 20 breaths per minute.
Deep, shallow breathing.
Both A and C.
Question 72
Which of the following strategies have led to a decrease in smoking prevalence in
Australia?
A.
B.
C.
D.
E.
Anti-smoking advertisements
Cigarette price
Nicotine replacement therapy
A and B
A, B and C
Question 73
Approximately what proportion of indigenous peoples live in "remote" and "very remote"
communities in Australia?
A.
B.
C.
D.
E.
Less than 1%
9%
27%
50%
78%
Question 74
Which of the following is NOT considered a parasomnia?
A.
B.
C.
D.
E.
Sleep terrors
Nocturnal epilepsy
Sleepwalking
Confusional arousal
Teeth grinding
Question 75
Which of the following is correct with regards to insomnia?
A.
B.
C.
D.
Insomnia is associated with sleep restriction.
Prevalence of insomnia is higher in males.
Insomnia can be treated with non-sedating antidepressants
Behavioural treatments such as cognitive behavioural therapy (CBT), relaxation and
circardian regulation are just as useful as pharmacologic treatments.
E. Development of insomnia is more likely in younger populations.
Question 76
What is sleep restriction and why is it used to treat insomnia?
A. Restraining a patient in a bed and holding them still to bore them into sleeping.
B. Preventing a patient from sleeping for 48 hours to 'reset' their circadian rhythm.
C. Instructing a patient to sleep through a whole day to allow them to catch up on lost
sleep.
D. Instructing a patient to get up if they cannot get to sleep and return to bed when they
are actually sleepy to prevent them from lying in bed stressing.
E. Only being in bed for sleeping and restricting other activities in bed (ie. reading, eating)
to associate good sleeping behaviours with being in bed.
Question 77
Which of the following symptoms is NOT indicative of an "atypical" pneumonia?
A.
B.
C.
D.
E.
Non-purulent cough
Upper respiratory tract symptoms
Myalgia
Sudden onset
Arthralgia
Question 78
Which of the following is a risk factor for severe pneumonia?
A.
B.
C.
D.
E.
Malignancy
Immunosuppression
Congestive Cardiac Failure
Chronic Lung Disease
All of the Above
Question 79
Which of the following is characteristic of pneumonia?
A.
B.
C.
D.
E.
Stony dullness heard on percussion of the chest.
Upper lobe hyperlucency on chest X-ray.
Hyper-resonance on percussion.
Increased tactile fremitus on palpation.
Tracheal deviation towards the affected side of the lung.
Question 80
Mrs Jones is a 63 year old who has presented to the emergency department with acute
onset dyspnoea and productive cough, worsening over the last 3 days. She describes that
recently she has been waking at night with drenching sweats and she feels very weak and
lethargic.Mrs Jones is MOST likely to be infected with which bacteria?
A.
B.
C.
D.
E.
Pseudomonas aeruginosa
Streptococcus pneumoniae
Legionella pneumophila
Staphylococcus aureus
Chlamydia pneumoniae
Question 81
The Bradford Hill criteria for causality are used to determine if an exposure causes a
disease, for example, if smoking causes lung cancer.Which of the following is NOT one of
the criteria?
A.
B.
C.
D.
E.
The exposure must precede the outcome.
The association must have been replicated by at least three studies.
The association is unlikely to be due to chance.
The association is unlikely to be due to bias.
The association is unlikely to be due to confounding.
Question 82
Cystathionine γ-lyase (CSE) and Heme Oxygenase-2 are important enzymes in carotid body
oxygen sensing. Which of the following species will increase with a shift from hypoxia to
normoxia?
A.
B.
C.
D.
E.
Carbon monoxide
Pyruvate
Ammonia
Hydrogen sulphide
L-Cysteine
Question 83
Which of the following is not an adaptive response to chronic hypoxia?
A. Increase in ventilation
B. Decreased peripheral blood flow
C. Increased cardiac output
D. Decreased cellular anaerobic metabolism
E. Increased pulmonary arterial pressures
Question 84
Peripheral oedema occurs in some patients with chronic obstructive pulmonary disease
(COPD). What is the primary mechanism for this oedema?
A. Failure of the left ventricle to maintan sufficient arterial pressure for adequate venous
return.
B. Increased thoracic pressures reducing cardiac output.
C. Increased pulmonary arterial pressure causing right ventricular hypertrophy and
failure.
D. Cytokines released in lung damage processes increase peripheral vascular permeability.
E. Decreased oxygen partial pressure causing peripheral vascular dilatation to ensure
adequate tissue oxygenation.
Question 85
Which of the following is not a feature of severe emphysema?
A.
B.
C.
D.
E.
Increased residual volume
Increased diffusion capacity
Possible reduction in vital capacity
Increased total lung capacity
Decreased FEV1/FVC ratio
Question 86
Oskar is a 56 year old man who has been a pack-a-day smoker since he was 22. He has been
complaining of a chronic cough and shortness of breath, and his doctor suspects he may
have emphysema. Which of the following is most affected in emphysema?
A.
B.
C.
D.
E.
lung tissue
upper airways
pleura
diaphragm
pulmonary blood vessels
Question 87
Which of the following changes occurs during the chronic stages of asthma?
A.
B.
C.
D.
E.
Decreased mucous production.
Permanent airway widening due to destruction of the airway wall.
Hyperplasia and hypertrophy of airway smooth muscle cells.
Increased mucociliary clearance.
Decrease in the number of submucosal glands.
Question 88
Which of the following is NOT involved in the sensitisation and acute stages of asthma?
A.
B.
C.
D.
E.
Antigen presentation to Th1 cells.
B cell class switching to produce IgE.
Antibodies to sensitizing antigen bind to mast cells.
Release of histamine, prostaglandins and leukotrienes.
Recruitment of eosinophils.
Question 89
Which one of the following is NOT an option for treatment of acute attacks or prevention in
asthma?
A.
B.
C.
D.
E.
Methacholine
Salbutamol
Fluticasone
Montelukast
Salmeterol
Question 90
Tracy is a 15 year old high school student who presents to you (her new GP) with a history
of increasing shortness of breath following exercise. Lung function testing reveals
bronchodilator reversibility and a provisional diagnosis of exercise-induced asthma is
made. Which of the following would NOT be considered a risk factor for asthma?
A.
B.
C.
D.
E.
Tracy having atopic eczema in the first year of life.
Tracy's mother and father both being asthma sufferers.
Tracy having an acute viral infection early in life.
Tracy's mother smoking cigarettes throughout her pregnancy.
Mutations in the lymphotoxin-α (aka TNF-β) gene AND associated atopy.
Question 91
Which of the following drugs is taken orally to treat asthma?
A.
B.
C.
D.
E.
Theophylline
Salmeterol
Tiotropium
Glycopyrronium
Montelukast
Question 92
Classification of asthma - what are the characteristics of persistent asthma?
A.
B.
C.
D.
E.
Attacks < 6 weeks apart
Use of β-agonists most days
Nocturnal asthma may occur
B and C
A and B
Question 93
Which of the following statements is FALSE regarding the smoking epidemic?
A.
B.
C.
D.
Anti-smoking advertising and price had the greatest impact on reducing rates.
Smoking is prohibited in high-roller casino rooms in NSW.
The hardening hypothesis was disproved by the ImpacTeen study.
Tobacco companies can still advertise indirectly through corporate social responsibility
programs.
E. In the late 19th century, a new fermentation process allowed for tobacco smoke to be
less harsh..
Question 94
Which of the following antibiotics is an example of a ?-lactam antibiotic?
A.
B.
C.
D.
E.
Quinupristin
Clindamycin
Levofloxacin
Meropenem
Vancomycin
Question 95
Which structures can be seen in the masticator space of the head?
A.
B.
C.
D.
E.
The pharyngeal venous plexus and the maxillary artery.
The parotid duct and gland.
Tensor veli palatini, levator veli palatini and the mastoid process.
Masseter, and the lateral and medial pterygoid muscles.
The common carotid artery, jugular vein and vagus nerve (CN X).
Question 96
You have just informed a young female patient that she has acute leukaemia. This is the
lady's first visit to you and she is present with a number of her family members. You wish
for her to stay at the hospital and immediately undergo chemotherapy but she remains
quiet and does not answer. Some of her family members aggressively ask you questions.
How do you respond to ensure that her decision making is not compromised?
A. Order her family to leave the room to ensure she decides autonomously and
independently.
B. Leave the room and allow them to make a decision whichever way they see fit.
C. Stay in the room, carefully see if their interactions seem to put pressure on the girl, but
allow them to make a decision as a group.
D. Stay in the room, carefully see if their interactions seem to put pressure on the girl, but
ensure that the final decision is made wholly by the girl.
E. Make the best decision for her based on your own clinical knowledge.
Question 97
In pharmacology, drugs are often released in different formulations. Glycopyrronium
bromide is currently prescribed as an inhaler to treat obstructive respiratory diseases, and
is also used intravenously during anaesthesia. What class of drug is glycopyrronium
bromide and for what pharmacological effects is it used?
A.
B.
C.
D.
E.
Anticholinergic; Bronchodilation, increasing secretions.
β-adrenergic agonist; Bronchodilation, hypertension.
Anticholinergic; Bronchodilation, reducing secretions.
β-adrenergic agonist; Bronchodilation, reducing secretions.
Phosphodiesterase (PDE) inhibitor; Bronchodilation, increasing secretions.
Question 98
Which of the following is cell-mediated rather than antibody-mediated?
A.
B.
C.
D.
E.
Type I hypersensitivity
Type II hypersensitivity
Type III hypersensitivity
Type IV hypersensitivity
Type V hypersensitivity
Question 99
Which of the follow is most likely to cause otitis media in a child?
A.
B.
C.
D.
E.
Streptococcus pneumoniae
Staphylococcus aureus
Staphylococcus epidermidis
Streptococcus pyogenes
Pseudomonas aeruginosa
Question 100
Mr Smith is has a chronic cough and you suspect he may have a vocal fold problem.Which
nerve supplies sensation to the vocal fold?
A.
B.
C.
D.
E.
Vagus nerve
Superior laryngeal nerve
Recurrent laryngeal nerve
Hypoglossal nerve
C1 through hypoglossal nerve
Question 101
In the young adult/teenage population within Australian indigenous communities, there is
an increasing burden of metabolic syndrome and youth alienation (self harm, violence,
substance abuse and mental abuse). Which of the following key changes to health policy
fails to address this trend?
A.
B.
C.
D.
E.
Gaining informed advice from community representatives and health workers.
Combining effort, rigour and resources to achieve the outcome.
Clearly identifying these issues and deciding on strategy.
Increased corporate investment.
Suppressing bureaucratic and ideological rhetoric aimed at these issues.
Question 102
Which of the following is TRUE about the respiratory system's response to increased
partial pressure of carbon dioxide in arterial blood?
A. Peripheral chemoreceptors in the medulla sense an increase in the partial pressure of
carbon dioxide in arterial blood.
B. Conditions of hypoxia increase the sensitivity of the ventilatory response to
hypercapnia.
C. Alcohol increases the sensitivity of the ventilatory response to hypercapnia.
D. The peripheral chemoreceptors are more sensitive to changes in arterial partial
pressure of carbon dioxide, rather than to changes in arterial partial pressure of oxygen.
E. Increased partial pressure of carbon dioxide in arterial blood above the normal range
stimulates hypoventilation in the subject.
Question 103
Which of the following will most likely result in the greatest increase in partial pressure of
oxygen in blood?
A.
B.
C.
D.
E.
Increasing the inspired oxygen pressure in a normal subject.
Increasing the inspired oxygen pressure in a patient with shunt.
Hyperventilation by a patient with a ventilation-perfusion mismatch.
Supplying oxygen to a cyanotic patient.
Increasing tidal volume in a patient with chronic obstructive pulmonary disease
(COPD).
Question 104
Which of the following are atypical pneumoniae?
A.
B.
C.
D.
E.
Pseudomonas aeruginosa.
Legionella species.
Haemophilus influenzae.
Chlamydia pneumoniae.
Klebsiella pneumoniae.
Question 105
You are the doctor in charge of a rural hospital. A young male patient has come to you in a
worryingly bad state. If you do not treat, he is likely to die within the week. You request
that he return the following morning but he does not return the next day, or the day after.
What should your response be?
A. Respect his right to choose his treatment for himself.
B. Enquire with nurses whether there was any reason for his absence, but do not contact
him directly as this goes against the privacy of the patient.
C. Ring the police and force the young man to return to hospital.
D. Ring the young man's home and family to see how he is doing and whether he
understands the gravity of the situation.
E. Read through the man's records to see if there was any reason for his absence, but do
not contact him.
Question 106
During swallowing, which muscle helps in opening the Eustachian tube?
A.
B.
C.
D.
E.
Levator veli palatini
Tensor veli palatini
Tensor tympani
Stapedius
Glossopharyngeus
Question 107
Which of the following is TRUE with regard to airway resistance?
A. Forced expiratory volume in one second (FEV1) provides an absolute measure of
airways resistance.
B. Airways resistance can be influenced by local reflex changes in arterial partial pressure
of carbon dioxide in order to maintain ventilation-perfusion matching.
C. Release of inflammatory mediators such as histamine causes smooth muscle relaxation
in small airways resulting in increased airways resistance.
D. Airways resistance is higher in the smaller airways due to higher total cross-sectional
area and laminar airflow.
E. With laminar flow, airways resistance is related to the radius of the airway to the fourth
power (r4).
Question 108
In what order does the respiratory system divide?
A. Trachea, lobar bronchi, principal bronchi, segmental bronchi, terminal bronchioles,
respiratory bronchioles, alveoli.
B. Trachea, principal bronchi, lobar bronchi, segmental bronchi, terminal bronchioles,
respiratory bronchioles, alveoli.
C. Trachea, primary bronchi, segmental bronchi, tertiary bronchioles, alveoli.
D. Trachea, principal bronchi, segmental bronchi, terminal bronchioles, alveoli.
E. Trachea, primary bronchi, secondary bronchi, terminal bronchioles, alveoli.
Question 109
What are the structural changes in the lung with age
A.
B.
C.
D.
E.
Decrease in elastic fibres
Increase in type 3 collagen
Decrease in number of alveoli
Increase in size of alveolar ducts
All of the above
Question 110
Which of the following degenerative changes occur in the alveolar sacs as they age?
A.
B.
C.
D.
E.
Elasticity decreases, compliance decreases and radial traction increases.
Elasticity increases, compliance increases and radial traction increases.
Elasticity decreases, compliance increases and radial traction decreases.
Elasticity increases, compliance decreases and radial traction increases.
Elasticity decreases, compliance increases and radial traction increases.
Question 111
Which of the following physiotherapy techniques/interventions is designed to help prevent
airway collapse and blockage?
A.
B.
C.
D.
E.
Postural drainage
Percussion
Hypertonic saline via nebuliser
Active Cycle of Breathing
Bubble PEP
Question 112
How does dimethyloxaloylglycine (DMOG) from the liver compensate for hyperoxiainduced retinal damage?
A. It negatively regulates prolyl hydroxylase which inhibits hypoxia-inducible factors,
leading to increased angiogenesis in the retina.
B. It enhances HIF production directly, leading to increased angiogenesis.
C. It positively regulates prolyl hydroxylase which inhibits hypoxia-inducible factors,
leading to increased angiogenesis in the retina.
D. It increases levels of glutathione, leading to oxidation of reactive oxygen species and
less retinal damage.
E. it negatively regulates glutathione levels, leading to increased levels of reactive oxygen
species and less retinal damage.
Question 113
a1-antitrypsin deficiency is a known genetic risk factor for developing chronic obstructive
pulmonary disease (COPD). ?1-antitrypsin utilises which mechanism to protect the lung
tissue from damage?
A. Inhibiting release of proteases by neutrophils by binding to a receptor on the neutrophil
cell wall.
B. Binding to neutrophil-released proteases directly and forming an inactive complex.
C. Inhibiting IL-8 signalling to block neutrophil infiltration of lung tissue.
D. Secreting neutrophil inhibitory factor preventing neutrophil adhesion to fibrinogen.
E. Suppressing IL-2 activation of alveolar macrophages.
Question 114
The two intraembryonic layers to differentiate during gastrulation are the:
A. ectoderm and endodern
B.
C.
D.
E.
epiblast and mesoderm
mesoderm and endoderm
ectoderm and hypoblast
epiblast and hypoblast
Question 115
A 55-year-old man with a history of peripheral vascular disease, who presents with a
complaint of a left foot ulcer and pain when walking short distances, is found to have a
popliteal stenosis and admitted for re-vascularisation. Four days after admission, on
postoperative day 3, he develops shortness of breath, hypoxia, and a productive cough.
Auscultation of his chest reveals decreased breath sounds at the lower aspect of the right
side of his chest. His morning leukocyte count is slightly higher than the day before at
11,000 cells/mL. An anterior-posterior bedside chest X-ray reveals right lower lobe
opacity.Which diagnosis will be most probable?
A.
B.
C.
D.
E.
Hospital-acquired pneumonia.
Atypical pneumonia.
Community-acquired pneumonia.
Atelectasis.
Lung cancer.
Question 116
Which muscle is able to retract the mandible?
A.
B.
C.
D.
E.
Masseter
Stylohyoid
Lateral pterygoid
Temporalis
Medial pterygoid
Question 117
John complains of being excessively sensitive to everyday noises. Which muscles do you
suspect are responsible?
A.
B.
C.
D.
E.
The helicis muscles
Stapedius
Tensor tympani
The helices muscles or stapedius
Stapedius or tensor tympani
Question 118
Which of the following structures is not connected to the hyoid bone by a muscle?
A.
B.
C.
D.
Mandible
Scapula
Styloid process
Sphenoid bone
E. Thyroid cartilage
Question 119
The parotid duct passes through which facial muscle?
A.
B.
C.
D.
E.
Masseter
Digastric
Medial Pterygoid
Lateral Pterygoid
Buccinator
Question 120
What is the function of the Zygomaticus major muscle?
A.
B.
C.
D.
E.
Lower the lip
Pull mouth together
Elevate mouth angle
Elevate the eyebrows
Tense the neck
Question 121
Which cranial nerve supplies motor function to the lateral rectus muscle?
A.
B.
C.
D.
E.
Abducens nerve (CN VI)
Oculomotor nerve (CN III)
Hypoglossal nerve (CN XII)
Facial nerve (CN VII)
Trochlear nerve (CN IV)
Question 122
What is a typical condition associated with the collapse of iridocorneal angle?
A.
B.
C.
D.
E.
Glaucoma
Cataract
Conjunctivitis
Impaired pupillary reflex
Vitreous detachment
Question 123
Which of the following are determinants of lung volume?
A.
B.
C.
D.
E.
Age
Sex
Height
Race
All of the above
Question 124
In a patient with severe interstitial lung disease, what happens to their diffusing capacity
and residual volume?
A.
B.
C.
D.
E.
Diffusing capacity is low and residual volume is low
Diffusing capacity is low and residual volume remains the same
Diffusing capacity is low and residual volume increases
Diffusing capacity is the same and residual volume decreases
Diffusing capacity is the same and residual volume increases
Question 125
Which of these structures should be visible when using an otoscope to observe a healthy
tympanic membrane?
A.
B.
C.
D.
E.
Internal acoustic meatus
Ampulla
Malleus
Round window
Stapes
Question 126
Which of the following anti-fungal drugs does NOT act by disrupting the phospholipid
bilayer of the cell membrane?
A.
B.
C.
D.
E.
Amphotericin B
Caspofungin
Fluconazole
Itraconazole
Voriconazole
Question 127
Which is the most common cause of infertility in men with a mutation in the cystic fibrosis
transmembrane conductance regulator (CFTR) gene?
A.
B.
C.
D.
E.
Decreased sperm motility.
Congenital absence of the vas deferens.
Low sperm count.
Development of autoantibodies to sperm which block movement and egg binding.
Hormonal disturbances resulting in low testosterone levels and hence infertility.
Question 128
The three steps to appraising an observational study are
A.
B.
C.
D.
E.
bias, selection, confounding
relevance, validity, results
relevance, validity and bias
blinding control
statistical control
Question 129
The physiological pH of blood is 7.4 (range 7.35-7.45). How do the intracellular pH levels
compare?
A.
B.
C.
D.
They are more acidic to facilitate removal of metabolic bases.
They are more acidic to facilitate removal of metabolic acids.
They are within the same range.
They are more alkaline to facilitate transport of metabolic acids to the intra-cellular
fluid.
E. They are more alkaline to facilitate transport of metabolic bases to the intra-cellular
fluid.
Question 130
Which of the following is the preferred position for a child to be in during a simple,
minimally invasive procedure?
A.
B.
C.
D.
E.
Supine surrounded by lots of smiling people.
Therapeutic position where the child is sitting on his or her parent's lap.
On their side, in recovery position with a teddy bear.
Upright in a sturdy chair.
Gently fastened to the bed so that they cannot wriggle away.
Question 131
What is open disclosure?
A.
B.
C.
D.
E.
An apology or expression of regret for a mistake in health care.
An opportunity for the patient, and their family and carers, to relate their experiences.
An explanation for why an adverse event occured in health care.
A and C.
A, B, and C.
Question 132
Which of the following statements is most correct about the normal sleep structure?
A.
B.
C.
D.
E.
A sleep cycle is between 150 to 180 minutes long.
Deep sleep occurs during the later stages of sleeping.
Rapid eye movement (REM) sleep phases become longer at the end of a period of sleep.
Deep sleep occurs throughout a period of sleeping.
Light and deep sleep occur intermittently throughout a period of sleep.
Question 133
Mr. Jones is a 35 year old male brought in by ambulance overnight after falling over due to
a dizzy spell. Upon further questioning, he mentions that he has a general tingling sensation
on his face along with difficulty blinking. Upon examination, you find out that he is unable
to raise his left eyebrow and unable to bare his teeth. Which cranial nerve is most likely
affected in Mr. Jones' case?
A.
B.
C.
D.
E.
V1
V2
V3
VII
XII
Question 134
In Australia, there is a large increase in the number of road traffic injuries when drivers
transition from learner permits to probationary licences. What is the most significant factor
contributing to this increase?
A.
B.
C.
D.
E.
Increased consumption of alcohol.
Increased driving in rural and remote areas.
Poor road design.
Inexperienced drivers now driving unsupervised.
Intentional risk taking by young drivers.
Question 135
Which of the following are a form of interstitial lung disease?
A.
B.
C.
D.
E.
Asthma
Hypersensitivity pneumonitis
Idiopathic pulmonary fibrosis
Pulmonary hypertension
Chronic obstructive pulmonary disease (COPD)
Question 136
A stapedectomy is a surgical procedure generally performed to treat which ear disease?
A.
B.
C.
D.
E.
Cholesteatoma.
> 3 episodes of acute otitis media in 6 months.
Sensory hearing loss.
Otosclerosis.
Traumatic perforation of the tympanic membrane.
Question 137
Which of the following lung irritants is the most common cause of chronic obstructive
pulmonary disease?
A.
B.
C.
D.
E.
Pollen
Air pollution
Cigarette smoke
Microbes
All of the above
Question 138
What is the FIRST step in the management of a tension pneumothorax with signs of
haemodynamic compromise?
A.
B.
C.
D.
E.
Needle thoracotomy in the 2nd intercostal space at the mid-clavicular line.
Chest X-ray to confirm clinical diagnosis.
Surgery to repair the damage to the pleura.
Chest drain in the 4th/5th intercostal space at the mid-axillary line.
Intubation to secure an airway.
Question 139
According to the International Agency for Research on Cancer, for which of the following
cancers is there insufficient evidence that smoking is a cause?
A.
B.
C.
D.
E.
Cervical cancer
Bladder cancer
Stomach cancer
Liver cancer
All of the above have sufficient evidence linking their incidence to smoking as a risk
factor.
Question 140
If a patient presents with dyspnoea, which of the following symptoms is associated with a
cardiac cause?
A.
B.
C.
D.
E.
Sputum
Wheeze
Cough
Haemoptysis
Palpitations
Question 141
The experience of trauma has unique features in particular sub-populations. Amongst
refugees, post-traumatic stress disorder (PTSD) is particularly prevalent because
A. Refugees lack the resilience and capacity to absorb the effects of normal hardship
B. PTSD is not particularly prevalent in refugee populations, because they experience
survivor elation
C. Refugees fear persecution and vilification in the countries into which they have fled
D. Refugees are unable to seek help because their experience of torture or persecution was
often enabled by medical professionals in their home country.
E. They are biologically predisposed to the development of PTSD in the face of normal
stressors.
Question 142
Mary Smith is a 70-year-old woman who has been admitted to hospital with an acute
exacerbation of airflow limitation on a long-standing background of chronic obstructive
pulmonary disease (COPD). She is commenced on oxygen therapy. Her arterial blood gases
on room air are measured and the results are as follows:pH 7.3Partial pressure of carbon
dioxide: 70 mmHgPartial pressure of oxygen: 60 mmHgBicarbonate concentration: 30
mmol/LWhat do these results demonstrate?
A.
B.
C.
D.
E.
Moderate hypoxemia and a partially-compensated respiratory acidosis.
Primary metabolic alkalosis with secondary respiratory acidosis.
The effects of alveolar hyperventilation.
The effects of low volumes of physiological dead space.
Uncompensated respiratory acidosis.
Question 143
What does the Abducens nerve (CNVI) innervate?
A.
B.
C.
D.
E.
Superior Oblique
Medial Rectus
Lateral Rectus
Superior Rectus
Levator Palpebrae Superior
Question 144
What is the sensory nerve supply to the anterior 2/3 of the tongue?
A.
B.
C.
D.
E.
CN V3 - Mandibular branch of Trigeminal Nerve
CN VII - Facial Nerve
CN IX - Glossopharyngeal Nerve
CN X - Vagus Nerve
CN XII - Hypoglossal Nerve
Question 145
Which of the following statements concerning the ventilation-perfusion ratio (V/Q ratio) is
true?
A.
B.
C.
D.
E.
If ventilation is kept constant and perfusion is increased, the V/Q ratio will increase.
If ventilation is increased and perfusion is increased, the V/Q ratio will stay constant.
If ventilation is decreased and perfusion is decreased, the V/Q ratio will decrease.
If ventilation is increased and perfusion is kept constant, the V/Q ratio will increase.
If ventilation is decreased and perfusion is increased, the V/Q ratio will stay constant.
Question 146
Which of the following statements regarding the oxyhaemoglobin dissociation curve is
true?
A. A right shift of the curve indicates an increase in the oxygen affinity for haemoglobin at
a fixed partial pressure..
B. The curve shows that the amount of oxygen bound to haemoglobin is directly
proportional to the partial pressure of oxygen.
C. A left shift of the curve indicates a decrease in the oxygen affinity for haemoglobin at a
fixed partial pressure.
D. The curve shows that the amount of oxygen carried by haemoglobin drops suddenly at
an oxygen concentration of 85 mmHg.
E. The steep lower portion of the curve shows that diffusion of oxygen into tissues is
possible with only small changes in oxygen partial pressure.
Question 147
Which of the following is true?
A. The invention of the cigarette-rolling machine dramatically increased the consumption
of cigarettes.
B. Cigarette rationing in the World Wars led to a greater increase in tobacco consumption
in men than women
C. The invention of ecigarettes has reduced tobacco use in young women
D. Less than half the retail price of cigarettes in Australia is due to tax
E. Plain packaging laws prohibit retailers from displaying cigarettes on open cabinets
Question 148
Which of the following statements regarding Cystic Fibrosis (CF) is most true?
A. Some heterozygous carriers of a CF gene can be found to have significant reduction of
CFTR activity, but not enough to warrant diagnosis of CF
B. Both ataluren and lumacaftor have been shown in trials to "chaperone" produced CFTR
to the cell membrane
C. Females with CF should be strongly encouraged to get assessed for their fertility
D. Carrying one severe mutation and one mild mutation will leave an individual to suffer
severe CF
E. Newborn screening helps to detect CF in over 95% of affected babies
Question 149
Ms Yee has a left parotid gland tumour, compressing the parotid gland and the structures
within it. Which clinical symptom might you expect Ms Yee have?
A.
B.
C.
D.
E.
A numb tongue.
Ptosis of the left eye.
A lop-sided smile.
An atrophied left temporalis muscle.
Tingling and pain on her forehead.
Question 150
Sally is a 5 year old girl who has come into ED complaining of breathlessness and wheeze
on a history of asthma. She has been given salbutamol and her breathlessness and wheeze
has since lessened.Which of the following is Sally also most likely to have?
A.
B.
C.
D.
Alveolar inflammation
Anaphylaxis
Urticaria
Vasculitis
E. Butterfly rash
Question 151
For which of the following would treatment with corticosteroids and/or
immunosuppressive therapy be least effective?
A.
B.
C.
D.
E.
Idiopathic pulmonary fibrosis
Cryptogenic organising pneumonia
Respiratory bronchiolitis - interstitial lung disease
Granulomatous interstitial lung disease
Nonspecific interstitial pneumonia
Question 152
A pharyngeal sinus or fistula may develop due to improper development of which
pharyngeal groove?
A.
B.
C.
D.
E.
The first pharyngeal groove
The second pharyngeal groove
The third pharyngeal groove
The fourth pharyngeal groove
The fifth pharyngeal groove
Question 153
Which of the following is not supplied by oculomotor nerve?
A.
B.
C.
D.
E.
The superior rectus muscle.
The levator palpebral superior muscle.
The ciliary body.
The lateral rectus muscle.
Sphincter pupillae.
Question 154
All of the following can be seen during a physical examination of a patient with interstitial
lung disease, with the EXCEPTION of:
A.
B.
C.
D.
E.
Inspiratory crackles
Wheezes
Clubbing
Pulmonary Hypertension
Hyper-resonant sounds
Question 155
A lung function test on a patient with interstitial lung disease will show a decrease in all of
the following parameters with the EXCEPTION of:
A. TLC
B. FVC
C. FEV1/FVC
D. DLCO
E. RV
Question 156
What is the soft palate comprised of?
A. The muscles: Palatoglossus, Palatopharyngeus, Tensor veli palatini and Levator veli
palatini. The uvula and the palatine bone.
B. The muscles: Palatoglossus, Palatopharyngeus, Tensor veli palatini and Levator veli
palatini. The uvula, the palatoglossal arch and the palatopharyngeal arch.
C. The muscles: Palatoglossus, Palatopharyngeus and Styloglossus. The uvula, the
palatoglossal arch and the palatopharyngeal arch.
D. The muscles: Palatoglossus, Palatopharyngeus and Styloglossus. The uvula, the
palatoglossal arch, palatopharyngeal arch, the maxilla and palatine bones.
E. The muscles: Palatoglossus, Palatopharyngeus, Tensor veli palatini and Levator veli
palatini. The palatoglossal arch and the palatopharyngeal arch.
Question 157
Which of the following is not a primary cause of clubbing?
A.
B.
C.
D.
E.
Interstitial Lung Disease
Infective Endocarditis
Chronic obstructive pulmonary disease (COPD)
Mesothelioma
Cirrhosis
Question 158
A variety of anatomical and physiological changes occur in the ageing lung. Which of the
follow is NOT an effect of ageing.
A.
B.
C.
D.
E.
Lung elastic recoil decreases
Airway closure decreases
Intrapleural pressures decrease in magnitude
The gradient between apical and basilar intrapleural pressures is maintained
Ventilation becomes decreasingly heterogeneous
Question 159
During inspiration the elastic tendency of the lungs to recoil pulls back against inflation
more strongly as overall lung volume increases. During upright normal breathing in a
healthy person the intrapleural pressure at all stages of breathing at the base of the lungs is
less negative than at the apex. What will be the consequences of the specific interplay
between these two forces during standing normal tidal breathing?
A. The bronchioles in apex of the lung will be open at rest and will ventilate more upon
inspiration.
B. The base of the lung will be held less inflated at rest and will have more capacity to
ventilate upon inspiration.
C. The surface tension in the alveoli at the base of the lung will be lower than in the apex.
D. The airflow in the base of the lung will be faster than the airflow in the apex.
E. The bloodflow at the apex of the lung will be greater than at the base.
Question 160
Which of the following explains the mechanism for expiration in normal, quiet breathing?
A. Use of abdominal muscle to depress ribs, creating a positive pressure in lungs and
causing air to leave lungs.
B. Diaphragm contracts, creating negative pressure in lungs and causing air to enter lungs.
C. Use of internal intercostal muscles to depress ribs, creating a positive pressure in lungs
and causing air to leave lungs.
D. Use of accessory muscles such as sternocleidomastoid to create a negative pressure in
lungs, causing air to enter lungs.
E. Elastic recoil of lungs creating a positive pressure in lungs, causing air to leave lungs.
Question 161
Which of the following pathological changes in the lungs does NOT result from smoking?
A.
B.
C.
D.
E.
Paralysis of the mucociliary escalator.
Inhibition of α1-antritrypsin in the lung parenchyma.
Squamous metaplasia of the airway epithelium.
Hyperplasia of airway wall goblet cells.
Lesions containing pronounced fibroblastic proliferation.
Question 162
Which of the following strategies would be most effective in decreasing public health costs
due to smoking?
A.
B.
C.
D.
E.
Selling lighter or milder cigarettes.
Advising people to only smoke twice in the day.
Education people's family and friends on throwing out their cigarettes.
Advise them to quit straight away.
Advise them to plan about quitting before they quit.
Question 163
Which of the following describes the hardening hypothesis for smoking?
A. Smoke that contains higher quantities of hard PCAHs is likely to be more detrimental to
health.
B. Increasing emphasis on denormalisation laws will ultimately reduce the prevalence of
smoking.
C. Of the populations with a lower prevalence of smoking, the remaining smokers are
relatively dependent and less likely to quit.
D. Substituting for cigarettes with lower nicotine quantities will simply result in the
smoker taking "harder" drags.
E. None of the above.
Question 164
Which of the following is not caused by smoking?
A.
B.
C.
D.
E.
Anhedonia
Bronchitis
Lung cancer
Type II Diabetes
Emphysema
Question 165
Regarding deaf children (with no other disability) who receive a cochlear implant before
the age of 18 months, what percentage learn to communicate using speech and receive
regular schooling?
A.
B.
C.
D.
E.
20%
50%
90%
95%
100%
Question 166
Salbutamol is a short-acting β-adrenergic agonist (SABA) whilst salmeterol is a long-acting
β-adrenergic agonist (LABA). Both produce bronchodilation. Which of the following
explains why salmeterol is longer acting?
A. Salmeterol is a larger molecule with a long lipid soluble tail so it takes longer to be
metabolized.
B. Salmeterol diffuses into the cell so it can't be metabolized by extracellular enzymes.
C. Salmeterol is typically given in higher doses so its bronchodilation effect lasts longer.
D. Salmeterol has a long lipid soluble tail which sits in the cell membrane, while the head
of the molecule activates the receptor.
E. Salmeterol is typically administered in combination with an inhaled corticosteroid so
its perceived effect is greater.
Question 167
A patient comes into your clinic and wishes to have her eye tested. If you ask her to abduct
her eye and look up, what muscle would you be testing?
A.
B.
C.
D.
E.
Superior oblique muscle
Inferior rectus muscle
Superior rectus muscle
Inferior oblique muscle
Medial rectus muscle
Question 168
In the presence of hypoxia, a wide alveolar-arterial (A-a) gradient would possibly indicate
which of the following?
A.
B.
C.
D.
E.
Pulmonary shunt.
Presence of interstitial lung disease.
Presence of chronic obstructive pulmonary disease (COPD)
Atelectasis.
All of the above.
Question 169
Narcolepsy is most typically described as excessive daytime sleepiness, combined with
which of the following?
A. Sleep onset in less than eight minutes, with rapid onset of deep (Stage N3) sleep.
B. Sleep onset in less than eight minutes, with rapid onset of rapid eye movement (REM)
sleep.
C. Sleep onset when excited, resulting in collapsing.
D. Inability to sleep at night, only able to sleep during the day.
E. Sleep onset less than eight minutes, with slowing and cessation of breathing.
Question 170
Grommets are a surgical treatment for which of the following?
A.
B.
C.
D.
E.
Ear wax build-up.
Acute otitis media.
Otitis media with effusion.
Perforated tympanic membrane.
Cholesteatoma.
Question 171
Spirometry assesses which of the following?
A.
B.
C.
D.
E.
Total lung capacity
Forced vital capacity
Tidal breath
Functional residual capacity
Residual volume
Question 172
In the context of surgery, smoking causes all except which of the following?
A.
B.
C.
D.
E.
Post-operative nausea and vomiting.
Delayed bone healing times.
Delayed wound healing times.
Increased wound infections.
Increased risk of anaesthesia.
Question 173
There are four parts to the "structure of smoking" that contribute to its addictive power.
These include all but which of the following?
A.
B.
C.
D.
E.
The context in which smoking occurs.
The ritual of smoking.
The sensory input obtained from smoking (i.e sound, taste, smell).
Nicotine stimulation.
The number of cigarettes smoked.
Question 174
In interstitial lung diseases (ILD), lung function tests most often show which of the
following?
A.
B.
C.
D.
E.
Reduced carbon monoxide diffusing capacity (DLCO)
Increased total lung capacity (TLC)
Airflow obstruction
Elevated arterial partial pressure of carbon dioxide
A decrease in the FEV1 to FVC ratio (forced expiratory volume in one second to forced
vital capacity)
Question 175
Which of the following muscles arises from the first pharyngeal arch?
A.
B.
C.
D.
E.
Stylohyoid
Stylopharyngeus
Mylohyoid
Palatopharyngeus
Levator veli palatini
Question 176
According to the Haddon Matrix, effective and timely emergency response is an example of
what type of factor for the prevention of road traffic injury and death?
A.
B.
C.
D.
E.
Post-crash environmental.
Post-crash social.
Pre-crash human.
Crash vehicle.
Crash environmental.
Question 177
Which of the following is false regarding the influences of spirituality on health?
A.
B.
C.
D.
E.
It is a core domain in assessing the quality of life in patients with cancer
It could interfere in a patient's acceptance of clinical information
It can contribute to longevity in patients with cancer
It can contribute to expectations of health staff
Empirical evidence suggests that spirituality is too subjective to be supported as having
protective effects on health
Question 178
Which of the following pathologies cannot be distinguished on a lateral X-ray of the head
and neck?
A.
B.
C.
D.
E.
Airway narrowing.
Sella turcica enlargement.
A blocked right maxillary sinus.
Sclerosis of the mastoid process.
Fracture of the mandible.
Question 179
Which structure is a boundary to both the anterior and posterior triangles of the neck?
A.
B.
C.
D.
E.
Angle of the mandible.
Sternocleidomastoid muscle.
Posterior belly of the digastric muscle.
Common carotid artery.
Clavicle.
Question 180
Which of the following activities improve children and families' experience in hospital?
A.
B.
C.
D.
E.
Continuous opportunities for play.
Self-esteem reinforcement.
Opportunities to express themselves with art.
Stress management techniques.
All of the above.
Question 181
The stimulus based model of stress proposes that stress is external to the person, who
experiences a reaction to the stress. Which of the following is NOT a weakness/limitation of
this model?
A.
B.
C.
D.
It treats the person as a passive respondent to a stressful event.
It does not recognise the person's role in creating the stressful event.
It undervalues the person's ability to moderate their response with effective coping.
It implies an internal locus of control whereby the person is unable to change the
situation.
E. It does not suggest an avenue to prevent the event occurring.
Question 182
Doubling alveolar ventilation will:
A.
B.
C.
D.
E.
double the PaO2
cause respiratory acidosis
have no effect
halve the PACO2
induce fainting
Question 183
Where does the mandibular branch of the Trigeminal nerve (CNV3) exit the skull?
A.
B.
C.
D.
E.
Foramen Ovale
Foramen Rotundum
Superior Orbital Fissue
Jugular Foramen
Internal Acoustic Meatus
Question 184
Which of the following is an important difference between salbutamol and salmeterol?
A.
B.
C.
D.
E.
Salbutamol and salmeterol act on different receptors.
Salmeterol has a slower speed of onset.
Salmeterol causes fewer systemic side effects.
Only salmeterol improves mucociliary clearance.
Only salmeterol inhibits the release of contractile mediators.
Question 185
Which of the following does not belong to the conducting zone of the respiratory system?
A.
B.
C.
D.
E.
The alveoli
The bronchioles
The nose
The pharynx
The mouth
Question 186
What is the innervation of the diaphragm?
A.
B.
C.
D.
E.
Through the phrenic nerve from the posterior rami of C3-C5 spinal levels.
Through the phrenic nerve from the anterior rami of C3-C5 spinal levels.
Through the vagus nerve.
Through the intercostal nerve.
Through the phrenic nerve from anterior rami of C1-C2 spinal levels.
Question 187
A 35 year old man is presenting to your clinic with acute onset of fever, night sweats and
productive cough with rusty sputum and mild chest pain. A culture and Gram stain of his
sputum shows the presence of a Gram-positive organism. Which antibiotic you will most
likely prescribe to him?
A.
B.
C.
D.
E.
Doxycycline
Azithromycin
Ciprofloxicin
Amoxycillin
Penicillin V
Question 188
When listening to the chest of a patient, bronchial or vesicular breath sounds may be heard.
In a patient with healthy lungs, what sort of breath sounds should be heard when
auscultating lung tissue, and which part of the lungs do the sounds originate from?
A.
B.
C.
D.
E.
bronchial, bronchi and large bronchioles
bronchial, small bronchioles and alveolar ducts
vesicular, trachea
vesicular, bronchi and large bronchioles
vesicular, small bronchioles and alveolar ducts
Question 189
A very large tumour of the parotid space will most likely impact upon the parapharyngeal
space by which mechanism?
A. Disrupting the junction of the parapharyngeal space with the danger space,
B. Projecting into the posterior aspect of the parapharyngeal space.
C. Invasion of the parotid-parapharyngeal border and subsequent compression of the
Vagus nerve (CN X).
D. Displacing the parapharyngeal space anteriorly into the carotid space.
E. Displacing the parapharyngeal space posteriorly into the masticator space.
Question 190
Which of the following is true in relation to the mediastinum?
A.
B.
C.
D.
E.
The pulmonary veins exit the lungs inferior to the pulmonary arteries.
The aorta arches posteriorly over the right main bronchus.
The pulmonary veins enter the hila at the level of the sternal angle.
The azygos vein arches over the right hilum, emptying into the superior vena cava.
The bronchi enter the left hilum anteriorly, because of the position of the heart.
Question 191
How many pairs of ribs attach solely to the manubrium of the sternum?
A.
B.
C.
D.
E.
0
1
2
3
4
Question 192
Which of the following is not a part of the contents in the mediastiunum?
A.
B.
C.
D.
The thymus.
The phrenic nerves.
The vocal folds.
The common carotid artery.
E. The carina of the trachea.
Question 193
With regard to health and health care, which of the following provides the most correct
reason why Indigenous children in Australian remote communities are severely
disadvantaged compared to non-Indigenous children?
A. 30% of the Indigenous population live urban/city centres.
B. They carry a high burden of infectious diseases, including respiratory illness,
gastroenteritis, ear infections and skin diseases.
C. They carry a high burden of chronic disease including diabetes mellitus, hypertension,
renal disease and vascular disease.
D. Growing antibiotic resistance has led to loss of efficient treatment for infectious disease.
E. There is no breast or cervical cancer screening.
Question 194
A 50 year old man visits his GP concerned about where or not he should undertake a PSA
screen for prostate cancer. You give him decision aids presenting evidence for the
outcomes of the PSA screen. Decision aids are most beneficial in:
A.
B.
C.
D.
E.
directing patients towards the right decision
improving patient knowledge
reducing consultation time length
better selecting who is more appropriate to screen
encouraging paternalistic decision making
Question 195
Caitlin is a 5 year old girl brought into the ED by her mother with shortness of breath and
audible wheeze. Her mother tells you that this has never happened before, but that her
older brother had a similar episode when he was 5 years of age that was discovered to be
asthma. What findings would allow for a potential diagnosis of asthma in Caitlin?
A. Positive sputum culture.
B. Increased forced vital capacity (FVC).
C. Decreased FEV1 to FVC ratio (forced expiratory volume in one second to forced vital
capacity).
D. More than 25% improvement in forced expiratory volume in one second (FEV1)
following bronchodilator inhalation.
E. Increased breath sounds on auscultation.
Question 196
What cranial nerve carries preganglionic autonomic fibres which synapse in the otic
ganglion?
A. Oculomotor nerve (CN III)
B. Trigeminal nerve (CN V)
C. Facial nerve (CN VII)
D. Glossopharyngeal nerve (CN IX)
E. Vagus nerve (CN X)
Question 197
Which of the following are important issues in medical certification?
A.
B.
C.
D.
E.
Patient honesty
Doctor honesty
Patient privacy
The doctor-patient relationship
All of the above
Question 198
Which of the following will be best corrected by hyperventilation?
A.
B.
C.
D.
E.
Shunt
Alkalosis
A defect in diffusion capacity
A reduced inspired oxygen percentage
Ventilation/perfusion mismatch
Question 199
The two main pathological features of asthma, bronchoconstriction and airway
inflammation are treated pharmacologically with different drugs. These include shortacting β-adrenergic agonists (SABA), long-acting β-adrenergic agonists (LABA),
phosphodiesterase (PDE) inhibitors, inhaled corticosteroids and anticholinergic
drugs.Which of the following are appropriate drug classes to treat: i) bronchoconstriction
and ii) inflammation.
A.
B.
C.
D.
E.
i) SABA ii) inhaled corticosteroid
i) SABA ii) PDE inhibitor
i) inhaled corticosteroid ii) anticholinergic
i) SABA ii) anticholinergic
i) LABA ii) PDE inhibitor
Question 200
There are a number of causes of bacterial pneumonia. Which of the following is preventable
through vaccination?
A.
B.
C.
D.
E.
Staphylococcus aureus
Klebsiella pneumoniae
Escherichia coli
Streptococcus pneumoniae
Legionella longbeachae
Question 201
What is not part of the primary survey for trauma examination?
A.
B.
C.
D.
E.
Circulation
Airway and C Spine
Blood glucose level
Breathing
Liver Function Tests
Question 202
What is the best explanation of sensitisation?
A. An allergen leaches through the epithelium, where antigen presenting cells pick up the
antigen, migrate to the nearest lymph node and are recognised by B cells, signalling
class switching to IgE antibodies
B. An allergen leaches through the epithelium, whereby a resident mast cell is stimulated
through the cross linking of antigens onto IgE antibodies, which then stimulates the
release of preformed and newly formed mediators.
C. An allergen leaches through the epithelium, which is then phagocytosed by resident
macrophages, mounting a local inflammatory response.
D. An allergen leaches through the epithelium, where antigen presenting cells pick up the
antigens and migrate to the nearest lymph node whereby T helper cells are activated
and encourage class switching in B cells to produce IgE antibodies.
E. None of the above
Question 203
You are a GP working in a busy practice. A patient, who you have seen for several years,
comes to see you and asks for a medical certificate. It is clear, however, that the patient is
not sick and it appears that she wants the certificate to avoid an exam. According to ethical
and professional guidelines, which of the following is the best course of action?
A.
B.
C.
D.
Write a certificate to preserve the patient's trust in you.
Write a certificate so that you can attend to other patients who may need your help.
Decline to write a certificate.
Enquire about her reasons for wanting the certificate and see if there are alternative
solutions to her problems. Explain that you cannot sign the certificate because of the
importance you place on honesty and the possible legal implications.
E. Refer her to a specialist.
Question 204
Which of the following nerve lesions is correctly listed with it's associated loss of function?
A. Damage to the glossopharyngeal nerve (CN IX) may cause loss of sensation in the
laryngopharynx.
B. Damage to the vagus nerve (CN X) may impair the ability to equalise pressure of the
auditory (eustachian) tube.
C. Damage to the glossopharyngeal nerve (CN IX) will impair the ability to abduct the
vocal folds.
D. Damage to the glossopharyngeal nerve (CN IX) is the most likely cause of dysphagia.
E. Damage to the recurrent laryngeal nerve (a branch of the vagus nerve; CN X) will impair
the ability to tense the vocal folds.
Question 205
Which of the following is correct in regards to exocrine secretion?
A. All exocrine secretion is by a merocrine or apocrine mechanism.
B. The faster fluid flows through a reabsorption duct, the greater the reabsorption of ions
from the fluid.
C. The potassium concentration in sweat is highest under low flow rates.
D. Reabsorption of ions takes place predominantly in the acinus of a sweat gland.
E. Tight junctions between cells restrict all ion flow between them.
Question 206
A 8 year old girl with cystic fibrosis has been in the hospital for the last week following a
chest infection. She has been very anxious during procedures. You are trying to complete a
procedure on her but she is uncooperative. Which of the following is the most appropriate
intervention?
A.
B.
C.
D.
E.
Leave her now and try again in an hour once she has calmed down
Coach her in stress management techniques
Do the procedure in her hospital bed
Have the parent restrain the child from moving
Give her a puzzle to assemble during the procedure
Question 207
Select the most correct option:
A. Spirituality may be beneficial to the patient on a personal level, but is not beneficial in
the health care setting as it has no impact on the patients physical state of health
(cardiovascular health, pain, etc)..
B. 'Suffering' refers to only physical and emotional pain experienced and the
consequences of this physical and emotional pain.
C. Spirituality can have a negative effect on health when it presents itself as an obstacle to
appropriate health care.
D. There is no link between spirituality/religion and health, and spirituality cannot
improve the physical health of patients.
E. Suffering is an all-encompassing societal and community experience which may then
have physical, emotional, psychological and other negative effects on an individual.
Question 208
Which of the following statements is INCORRECT regarding shift work and circadian
disruption?
A. Shift workers are more likely to experience weight loss due to increased disruption in
their eating habits.
B. Shift workers can experience reproductive difficulties such as decreased fertility and
increased risk of miscarriage.
C. Consequences of shift work and circadian disruption include impaired concentration
and reaction times.
D. The number of fatigue-related car crashes in Australia is similar to that of alcoholrelated car crashes.
E. Shift work sleep disorder is characterised by difficulty sleeping during the day, and
difficulty staying awake at night.
Question 209
Which of the following is considered the strongest treatment for circadian disruption?
A.
B.
C.
D.
E.
Bright light exposure
Low temperature
Caffeine
Orally administered melatonin
Dim light exposure
Question 210
In patients with chronic obstructive pulmonary disease (COPD), which volume increases
the most in relative proportion?
A.
B.
C.
D.
E.
Total lung capacity.
Vital capacity.
Tidal volume.
Residual volume.
Functional residual capacity.
Question 211
Ben is a medical intern who is working night shift. In order to delay the onset of tiredness
and sleepiness, what would you recommend Ben do?
A.
B.
C.
D.
E.
He should take a nap before his night shift, of 30-60 minutes duration.
He should take regular naps during his shift as required.
He should drink coffee before his shift starts.
He should take a nap before his shift, of less than 30 minutes duration.
He should take melatonin supplements before his shift starts.
Question 212
You are working in a rural Chinese community for an elective. Which of the following is
becoming a greater cause of death today in developing countries such as China compared
to 20 years ago?
A.
B.
C.
D.
Lower respiratory tract infections
Malnutrition
Preterm birth complications
Ischemic Heart Disease
E. Tuberculosis
Question 213
In fibrosis of the lung there may be excessive radial traction on the airways. This leads to
A.
B.
C.
D.
E.
collapse of the small airways
enlarged airway calibre related to lung volume
increased forced vital capacity
low FEV1/FVC ratio
all of the above
Question 214
Which of the following statement most correctly describes developmental changes in
sleep?
A.
B.
C.
D.
E.
In the first year of life, sleep often starts with non-rapid eye movement (NREM) sleep.
Rapid eye movement (REM) sleep stages emerge over first 2 - 6 months of life.
Slow wave sleep (SWS) develops in young chidren and increases with age.
Arousals increase in frequency with age, so sleep is less consolidated.
Less wake after sleep onset (WASO) as age increases.
Question 215
From which of the following structures does the external auditory canal develop?
A.
B.
C.
D.
E.
The 1st branchial groove.
The 2nd branchial groove.
The 3rd branchial groove.
None of the above.
All of the above.
Question 216
Which cranial nerve is responsible for sensory innervation of the middle ear?
A.
B.
C.
D.
E.
The mandibular nerve (CN V3)
The abducens nerve (CN VI)
The facial nerve (CN VII)
The oculomotor nerve (CN VIII)
The glossopharyngeal nerve (CN IX)
Question 217
A 20year old female patient with Cystic Fibrosis is admitted to hospital with an acute
infective exacerbation. On your multidisciplinary ward round, she asks the physiotherapist
in your team about the long term efficacy of airway clearance techniques. You have recently
read the Pryor 2006 and McIlwaine 2003 studies looking at this question and tell her that
most of the interventions have no significant difference in exacerbations except for one.
Which physiotherapy intervention has been associated with a significant increase in
exacerbations compared to the others over 1 year?
A.
B.
C.
D.
E.
Positive expiratory pressure
High frequency chest wall oscillation vest
Autogenic Drainage
Flutter device
Active Cycle of Breathing technique
Question 218
Iron (Fe) deficiency (serum ferritin
A.
B.
C.
D.
E.
Narcolepsy
Obesity hypoventilation syndrome (hypercapnic OSA)
REM behaviour disorder
Restless legs syndrome (RLS)
Insomnia
Question 219
If you are an unaffected sibling of a child who has cystic fibrosis (CF) then what are the
chances that you are a carrier?
A.
B.
C.
D.
E.
0 (No chance)
1/4
1/2
2/3
1 (Certain)
Question 220
Opening of the vocal folds is essential for allowing normal and forced respiration. Which
muscle and associated nerve are responsible for opening the vocal folds in this manner?
A.
B.
C.
D.
E.
Lateral cricoarytenoideus and the superior laryngeal nerve.
Posterior cricoarytenoideus and the superior laryngeal nerve.
Lateral cricoarytenoideus and the recurrent laryngeal nerve.
Posterior cricoarytenoideus and the recurrent laryngeal nerve.
Cricothyroideus and the superior laryngeal nerve.
Question 221
What is the sensory, motor and taste nerve supply to the tongue? Pick the correct answer.
A. Sensory is V3 (mandibular) anteriorly, IX (Glossopharyngeal) posteriorly. Motor is XII
(hypoglossal) except palatoglossus X (vagus). Taste is XII hypoglossal
B. Sensory is V3 (mandibular) anteriorly, IX (Glossopharyngeal) posteriorly. Motor is XII
(hypoglossal). Taste is VII anteriorly (facial) and IX posteriorly (Glossopharyngeal)
C. Sensory is V3 (mandibular) anteriorly, IX (Glossopharyngeal) posteriorly. Motor is XII
(hypoglossal) except palatoglossus X (vagus). Taste is VII anteriorly (facial) and IX
posteriorly (Glossopharyngeal)
D. Sensory is XII anteriorly (hypoglossal) and IX (glossopharyngeal) posteriorly. Motor is
XII (hypoglossal) except palatoglossus X (vagus). Taste is XII hypoglossal
E. Sensory is XII anteriorly (hypoglossal) and IX (glossopharyngeal) posteriorly. Motor is
XII (hypoglossal). Taste is XII hypoglossal anteriorly and IX posteriorly
(glossopharyngeal).
Question 222
The ethmoid bone contains which concha? Select the most correct answer.
A.
B.
C.
D.
E.
superior concha
middle concha
superior, middle and inferior concha
middle and inferior concha
superior and middle concha
Question 223
A 60 year old man presents with a 2-year history of right knee pain. He has been
prescribed a non-selective NSAID, which he takes regularly but has not helped with his
pain and he has become very distressed. You recommend a knee arthoscopy, knowing
there is no evidence for the treatment, but in hopes that it will provide him relief. This is an
example of what type of cognitive error?
A.
B.
C.
D.
E.
Anchoring heuristic
Commision bias
Outcome bias
Latent error
Error in execution
Question 224
Many molecules are released from mast cells during an immune response. A number of
these molecules are derived from a membrane phospholipid precursor and therefore
synthesised de novo when the mast cell is activated. Which of the following molecules is
NOT derived from a membrane phospholipid precursor?
A.
B.
C.
D.
E.
Prostaglandin D2 (PG D2)
Leukotriene B4 (LT B4)
Platelet Activating Fator (PAF)
Leukotriene C4 (LT C4)
Eosinophil Chemotactic Factor (ECF)
Question 225
A patient presents to you with obstructive airway disease. They are unable to fully expire
to a normal residual volume, and their lungs are constantly hyperinflated. They are still
able to breathe in a normal tidal volume using their inspiratory reserve volume however
they are showing signs of severe peripheral and central hypoxia. What is the most likely
reason for this to occur?
A. The air is only ventilating the most superior lobes of the lungs which have poorer blood
flow.
B. The air will not make it past the conducting dead spaces into the alveoli, and thus blood
is not oxygenated.
C. The higher intrathoracic pressure will prevent venous return to the heart from the
inferior vena cava.
D. The higher intrathoracic pressure will push on the heart and prevent it from beating
fully.
E. The higher intrathoracic pressure will push on the pulmonary arteries and prevent
perfusion of the lungs.
Question 226
Short acting ?-agonists (SABAs), which are used to alleviate an acute exacerbation of
asthma, have a number of physiological effects within the airways. Which of the following is
the most clinically relevant mechanism of action of SABAs?
A.
B.
C.
D.
E.
They directly inhibit mast cell mediator release.
They improve mucociliary clearance by increasing frequency of cilia beating.
They relax airway smooth muscle to increase airway latency.
They decrease hypersensitivity of airways.
All of the above actions are equally important.
Question 227
Which of the following are not smokefree areas?
A.
B.
C.
D.
E.
Restaurants
Workplaces
Public transport
Casino high-roller rooms
None of the above
Question 228
A 55 year old male smoker presents to the emergency room with wheezing and shortness
of breath. He explains that he was at his work as a spray painter, and he removed his mask
and forgot to put it back on before beginning to paint again in a small enclosed space. You
give him bronchodilators and he improves immensely. What do you think he has?
A.
B.
C.
D.
E.
Occupational induced asthma caused by sensitisation
Pneumonia, which was exacerbated by the irritant exposure
Occupational induced asthma caused by irritants
COPD, this is probably an exacerbation of his long term symptoms due to smoking
Asbestosis, he has probably been exposed at work to asbestosis for a long time
Question 229
A patient comes to the emergency room with broken ribs due to falling off her bike. You
take a chest X-ray and notice, as well as the broken ribs, fibrotic nodular appearance on the
upper zones of her X-ray and enlarged hilar nodes. She tells you upon questioning that she
works as a sandblaster. What do you think she has?
A.
B.
C.
D.
Asbestosis, this X-ray is typical of a patient with asbestosis
Silicosis, however it is abnormal that she has had no respiratory symptoms
Silicosis, it is not unusual that she has had no respiratory symptoms
Lung cancer, this very rarely presents with respiratory symptoms until very late in the
disease and is associated with a poor prognosis
E. Metal fume fever, the fever probably contributed to her becoming unbalanced and
falling off her bike
Question 230
You're following an intern around at clinical school during rounds. Afterwards they
mention they're worried that their registrar is often quick to accept a suggested diagnosis
and regularly fails to complete any form of physical exam. It is possible this is due to a
cognitive bias in the registrar's diagnostic reasoning. Which of the following best describes
this type of bias?
A.
B.
C.
D.
E.
Premature closure
Omission bias
Outcome bias
Anchoring heuristic
Availability heuristic
Question 231
An 'anchoring heuristic' is identified as a type of cognitive error in diagnostic reasoning. It
relates to a tendency to fixate on first impressions and selected symptoms or signs or
simple investigation results are used as predictors of specific diagnosis.Which of the
following is the best strategy to minimise this form of bias?
A. Verify prevalence based on proper statistics.
B. Think beyond the most favoured diagnosis; reconsider in light of new data or
unexpected course of illness.
C. Consider the worst case scenario that may result from inaction.
D. Be realistic in expectations for therapeutic success; do not offer treatment in futile or
near futile situations.
E. Ask "Am I treating the patient, or myself?"efer her to a specialist.
Question 232
Which of the following is an example of an appropriate use of antibiotics?
A. They are continued when viral aetiology is confirmed and bacterial infection is unlikely.
B. They are given in dosages which produce low levels in blood long enough to eradicate
the bacteria.
C. Broad-spectrum antibiotics are used for the shortest duration.
D. Use of narrow spectrum β-lactam antibiotics for a lung infection.
E. Prescribing β-lactams for an infection by an atypical microbe.
Question 233
Which of the following is not an explanation for the current increase in demand for
emergency health services?
A. Australia's ageing population
B. The convenience of presenting to the emergency department
C. The opportunity for patients to be referred following admission to the emergency
department
D. Staff in the emergency department tend to work faster and more effectively than
primary care physicians
E. The increasing prevalence of chronic diseases
Question 234
Children, particularly infants, have a number of differences in their respiratory system
compared to adults. Which of the following is NOT one of those differences?
A.
B.
C.
D.
E.
A softer trachea.
A relatively large tongue compared to the size of their mouth.
Breathing which predominantly occurs using abdominal muscles.
Less compliant chest walls.
All are differences noted in the respiratory system of children.
Question 235
Which of the following is true in regards to the Pharyngeal arches?
A. There are 4 main arches with the fifth and sixth arches being rudimentary.
B. The first arch gives rise to the incus, malleus, maxilla, zygomatic, vomer, mandible,
temporal (squamous part) bones.
C. Only the third arch gives rise to the hyoid bone.
D. A and B.
E. A, B and C.
Question 236
How do prolyl hydroxylases act in response to reactive oxygen species?
A. They act to promote the production of glutathione, which is oxidised in order to protect
cells from reactive oxygen species.
B. They are negatively regulated by dimethyloxaloylglycine (DMOG), which leads to
decreased stability of hypoxia-inducible factors (HIFs).
C. They decrease the stability of hypoxia-inducible factors (HIFs), negatively regulating
this pathway.
D. They increase the stability of hypoxia-inducible factors (HIFs), positively regulating this
pathway.
E. They inhibit pyruvate dehydrogenase, leading to the conversion of pyruvate to lactic
acid.
Question 237
Which of the following is not a widely used way of reporting outcomes from prognostic
studies?
A.
B.
C.
D.
E.
% outcome at a point in time e.g. 5 year survival rates
medial survival
survival curves e.g. Kaplan Meier
relative risk
Hazard ratio
Question 238
Polio is still endemic to which of the following countries?
A.
B.
C.
D.
E.
China
Australia
Ethiopia
Afghanistan
Papua New Guinea
Question 239
Which of the following is an example of a passive safety measure for the prevention of road
traffic injuries?
A.
B.
C.
D.
E.
A public awareness campaign encouraging drivers to take frequent breaks.
Increased penalties for driving offences during holiday periods.
Speed cameras on busy roads.
Random breath testing by police.
Installation of air bags in all new cars.
Question 240
Pulmonary rehabilitation is one of the most effective strategies for long term management
of chronic obstructive pulmonary disease (COPD). This includes exercise training, which is
associated with improved exercise capacity and decreased dyspnoea. Which of the
following describes one of the changes which occurs in exercising skeletal muscles?
A.
B.
C.
D.
E.
Increased oxidative enzyme concentration.
Decreased density of mitochondria.
Increased myoglobin.
A and C.
A, B and C.
Question 241
What is the function of the transversus thoracis muscles?
A.
B.
C.
D.
E.
To raise the ribs during inspiration.
To pull the sternum back during inspiration.
To depress the ribs during expiration.
To support the posterior thoracic wall.
To pull the sternum back during expiration.
Question 242
Which of the following is the least likely explanation for swollen ankles?
A.
B.
C.
D.
E.
Pregnancy
Pleural effusion
Pulmonary oedema
Cirrhosis of the liver
Hypothyroidism
Question 243
What are the 4 main asbestos related diseases?
A.
B.
C.
D.
E.
Lung cancer, Asthma, COPD , Hypersensitivity pneumononitis
Asbestosis, Pneumonia, COPD, Lung cancer
Asthma, Lung cancer, Asbestosis, COPD
Mesothelioma, Lung cancer, Benign pleural disease, Asbestosis
Asbestosis, Mesothelioma, Asthma, Lung cancer
Question 244
Which of the following conditions would be considered most urgent for triage purposes?
A.
B.
C.
D.
E.
An 80 year old female with suspected Stage IV pancreatic cancer.
A 22 year old male with a distal phalanx dislocation.
A 42 year old female with a blood glucose of 1.9 mM.
A 18 year old who passed out and has a GCS of 8.
A 33 year old who inhaled a tic tac with slight wheeze.
Question 245
The semicircular canals connect to the vestibule by how many openings?
A.
B.
C.
D.
E.
2
3
4
5
6
Question 246
What constitutes the respiratory zone of the lungs?
A.
B.
C.
D.
E.
The area from the primary bronchi to the alveolar sacs.
The area from the trachea to the terminal bronchioles.
The area from the terminal bronchioles to the alveolar sacs
The area from the respiratory bronchioles to the avleolar sacs
The area from the alveolar ducts to the alveolar sacs
Question 247
Which of the following is correct regarding perfusion and gas transfer in the lungs?
A. Diffusing capacity of the lung is affected primarily by its elastic recoil.
B. Simple spirometry can be used to calculate a person's total lung capacity.
C. Carbon dioxide is a suitable gas to be used when measuring gas diffusion within the
lungs because it is more soluble in blood than oxygen is.
D. Methane or helium are suitable gases to be used when measuring gas diffusion in the
lungs because they do not diffuse into the alveolar capillaries.
E. Methane or helium are NOT suitable gases to be used when measuring gas diffusion in
the lungs because they do not diffuse into the alveolar capillaries.
Question 248
The MOST likely cause of a spontaneous pneumothorax in an otherwise healthy 17 year old
male is:
A.
B.
C.
D.
E.
Emphysema
Blunt trauma
α1-antitrypsin deficiency
Rupture of an apical bleb
Cystic fibrosis
Question 249
The foramen caecum of the adult tongue is:
A. the site of embryonic evagination of the thyroid gland
B. the point of division of the tongue into the anterior two-thirds and the posterior onethird
C. the site of embryonic evagination of the thymus gland
D. never observed in an adult human
E. a pathology of thyroid development
Question 250
The primitive streak is:
A.
B.
C.
D.
E.
derived as an extension of the cloacal membrane
formed by the folding of the hypoblast
the notochord which extends rostrally during gastrulation
formed during gastrulation due to epiblast cell movement to form the mesoderm
initiated by neural plate signalling
Question 251
The most common mutation in cystic fibrosis is ?F508, which affects the transporter called
cystic fibrosis transmembrane conductance regulator (CFTR). How does it do this?
A.
B.
C.
D.
E.
By preventing initial synthesis of the transporter.
By inhibiting processing of the protein into a mature transporter.
By affecting regulation of the transporter.
By altering the conductance of the transporter.
By reducing synthesis of the transporter.
Question 252
Which of the following findings would be most expected in advanced silicosis?
A. Pleural plaques on a chest X-ray.
B. Increased rheumatoid factor measured in the blood.
C. An FEV1 to FVC ratio of 50% (forced expiratory volume in one second to forced vital
capacity).
D. Fibrotic nodules with concentric collagen and birefringent particles within the lungs.
E. A family history of atopy.
Question 253
A patient presents with fine inspiratory crackles. What is one likely differential diagnosis?
A.
B.
C.
D.
E.
Asthma
A cold
Pulmonary fibrosis
Cancer
Consolidation
Question 254
A patient presents to you with dyspnoea. On examination, breath sounds are absent in the
left lung base, there is reduced vocal resonance on the left side and stony dullness is heard
on percussion. What is one likely diagnosis?
A.
B.
C.
D.
E.
Consolidation of the lung
Pleural effusion
Chronic obstructive pulmonary disease (COPD)
Interstitial lung disease
Pneumothorax
Question 255
A 56-year-old male patient presents with breathlessness and bilateral basal crepitations on
auscultation. Lung function tests reveal a decrease in total lung capacity and a decrease in
diffusing capacity for carbon monoxide (DLCO). Which of the following is the most likely
diagnosis?
A.
B.
C.
D.
E.
Chronic bronchitis
Idiopathic pulmonary fibrosis
Cystic fibrosis
Allergic bronchopulmonary aspergillosis
Chronic obstructive pulmonary disease (COPD)
Question 256
A 40 year old woman is seen in the office complaining of shortness of breath in climbing a
flight of stairs, which has been progressive over the past four months. Her physical
examination is unremarkable. Her chest X-rays show bilateral hilar lymphadenopathy, with
a mild diffuse infiltrate in the lung fields. Which of the following is the most likely
diagnosis?
A.
B.
C.
D.
E.
Interstitial pulmonary fibrosis (IPF)
Sarcoidosis
Wegener's granulomatosis
Asbestosis
Tuberculosis
Question 257
A 43-year-old woman with xerostomia, and keratoconjunctival and vaginal sicca (mouth,
eye, and vaginal dryness). She is complaining of shortness of breath, dry cough, and fine
crackles upon auscultation. She has low levels of oxygen saturation on examination.
Further investigations show that her total lung capacity is less than 80%. High-resolution
computed-tomography scanning shows symmetrical, lower-lobe, ground glass opacity, and
signs of traction bronchiectasis, with no honeycombing. Her serum tests positive for
extractable nuclear antigens (ENA), anti-nuclear antibodies (ANA) and rheumatoid factor
(RF).Which of the following is the most likely diagnosis?
A.
B.
C.
D.
E.
Idiopathic pulmonary fibrosis
Sarcoidosis
Connective tissue disease
Non-specific intersitial pneumonia
Lymphocytic interstitial pneumonia
Question 258
Rinne and Weber tests are performed on an patient with suspected hearing loss. The
results are as follows:RinneRight ear: air conduction exceeds bone conductionLeft ear: air
conduction exceeds bone conductionWeber: Sound is heard better in the right ear.What is
the most likely diagnosis?
A.
B.
C.
D.
E.
Sensorineural hearing loss in the right ear.
Sensorineural hearing loss in the left ear.
Conductive hearing loss in the right ear.
Conductive hearing loss in the left ear.
Normal hearing.
Question 259
Rinne and Weber tests are performed on a patient with suspected hearing loss. The results
are as follows:RinneRight ear: bone conduction exceeds air conductionLeft ear: air
conduction exceeds bone conductionWeber: Sound is heard more clearly in the right
earWhat is the most likely diagnosis?
A.
B.
C.
D.
Sensorineural hearing loss in the right ear.
Sensorineural hearing loss in the left ear.
Conductive hearing loss in the right ear.
Conductive hearing loss in the left ear.
E. Normal hearing.
Question 260
Which of the following conditions is not seen with Cystic fibrosis?
A.
B.
C.
D.
E.
Nasal polyposis
Focal biliary cirrhosis
Xerostomia
Cholestasis
Meconium ilieus
Question 261
Which of the following characteristics do NOT contribute to disease in Cystic fibrosis?
A.
B.
C.
D.
E.
decrease in airway Cl secretion due to impaired Calcium dependent Cl channel
Increase in Sodium reabsorption
Increase in production of pro-inflammatory cytokine
decrease in muco-ciliary clearance
increase in mucous production and viscosity
Question 262
Your long-term patient Friedrich has just died from a respiratory infection associated with
his cystic fibrosis. Which of the following respiratory pathogens is responsible for the
highest mortality from lung infections in adults with cystic fibrosis?
A.
B.
C.
D.
E.
Staphylococcus aureus
Pseudomonas aeruginosa
Haemophilus influenzae
Methicillin-resistant Staphylococcus aureus
Burkholderia cenocepacia
Question 263
Which one of the following is not a pathogen that commonly infects patients with cystic
fibrosis?
A.
B.
C.
D.
E.
Staphylococcus aureus
Bacillus anthracis
Haemophilus influenzae
Burkholderia cepacia
Pseudomonas aeruginosa
Question 264
What type of lung crackles are you most likely to hear on auscultation of a patient with
asbestosis?
A. Fine crackles on early inspiration and expiration
B. Course crackles on early to mid inspiration
C. Course crackles on late inspiration
D. Fine crackles on mid to late expiration
E. Fine crackles on mid to late inspiration
Question 265
Hypoxia-inducible factors (HIFs) mediate the body's response to inadequate oxygenation.
In the absence of enough oxygen to meet demand, which of the following enzymes directly
facilitates the switch from aerobic respiration to glycolysis?
A.
B.
C.
D.
E.
Prolyl hydroxylase
Asparagine hydroxylase
Pyruvate dehydrogenase
Pyruvate dehydrogenase kinase
Glutathione
Question 266
When viewing a chest X-ray you notice hyper-expanded lung fields and flat hemidiaphragms. What would you consider the best diagnosis based on these signs?
A.
B.
C.
D.
E.
Lung cancer
Bronchiectasis
Emphysema
Tuberculosis
Interstitial lung disease
Question 267
Which of the following scenarios would most likely cause respiratory alkalosis to be
diagnosed after appropriate investigations?
A.
B.
C.
D.
E.
A patient investigated after 5 minutes of hypoventilation.
A patient with a hepatocarcinoma causing hyper-secretion of bicarbonate.
A patient investigated after 5 minutes of hyperventilation.
A patient with chronic obstructive pulmonary disease (COPD).
A patient with Bowen's disease.
Question 268
What is the most common CF mutation in caucasian populations?
A.
B.
C.
D.
E.
p.F508Δ
p.G542X
p.G551D
p.R117H
p.A445E
Question 269
Jenny is 2 days old and was born premature. She has a rare genetic condition in which she
is unable to produce Prolyl Hydroxylase and Asparagine Hydroxylase. Her mother comes to
you worried and asks if Jenny will end up with any vision problems from the high-flow
oxygen therapy you have given her.Considering only their roles in oxygen sensing, what
will you tell Jenny's mother regarding her mutations?
A. She will be fine because we are also treating her with dimethyl oxaloylglycine (DMOG).
B. Prolyl hydroxylase and Asparagine hydroxylase serve to deactivate hypoxia-inducible
factors (HIF) so angiogenesis will not be retarded.
C. Hypoxia-inducible factors (HIF) will retard angiogenesis in the absence of prolyl
hydroxylase and asparagine hydroxylase
D. Prolyl hydroxylase and asparagine hydroxylase liberate Nrf2 which is protective
against reactive oxygen species (ROS), so this will result in ROS damage of her retinas.
E. In the absence of prolyl hydroxylase and asparagine hydroxylase, NADPH oxidase will
become hyperactive, producing damaging reactive oxygen species.
Question 270
What two muscles make up the faucial arches and what are their innervations?
A.
B.
C.
D.
E.
Palatoglossus and Palatopharyngeus (CNIX and CNX respectively)
Tensor veli palatini and Palatoglossus (CNV3 and CNX respectively)
Palatopharyngeus and Levator veli palatini (both supplied by Vagus [CNX])
Tensor veli palatini and Levator veli palatini (CNV3 and CNX respectively)
Palatoglossus and Palatopharyngeus (both supplied by Vagus [CNX])
Question 271
Which of the following is FALSE regarding antimicrobial therapy in lung infections?
A. β-lactam resistance in Streptococcus pneumoniae is incremental.
B. Mycoplasma sp. do not have cell walls and are therefore not destroyed by β-lactam
antibiotics.
C. β-lactam resistance in Haemophilus influenzae is incremental relative to levels of βlactamase the microbe produces.
D. Aminoglycosides are most useful for aerobic gram negative bacteria.
E. The 'societal' approach to antibiotic prescription is one that favours narrow spectrum
therapy.
Question 272
Nail clubbing is NOT associated with which of the following conditions?
A.
B.
C.
D.
E.
Idiopathic pulmonary fibrosis
Mesothelioma
Cystic fibrosis
Chronic obstructive pulmonary disease
Asbestosis
Question 273
What histological findings of the lung would you expect to see for a individual presenting
with chronic shortness of breath and was a sand-blaster for many years?
A. Well formed alveoli with minimal interstitial fibrosis.
B. Small stellate shaped scars around alveoli containing collagen fibres and dark coloured
macrophages.
C. Nodular lesions with concentric layers of collagen and birefringence under polarised
light.
D. Noncaseating granulomas with numerous giant cells around lymphatics and bronchi.
E. Diffuse interstitial fibrosis with golden beaded rods.
Question 274
Which of the following are correct with regards to sleep disorders?
A. Narcolepsy presents more often in the elderly, and presents with persistent daytime
sleeping.
B. Cataplexy can be treated with medications that suppress rapid eye movement (REM)
sleep, like tri-cyclic antidepressants (TCAs), selective serotonin reuptake inhibitors
(SSRIs) and serotonin-noradrenaline reuptake inhibitors.
C. Idiopathic hypersomnolence is an important differential for narcolepsy, and presents
with mild EDS (excessive daytime sleepiness).
D. Rapid eye movement (REM) sleep behaviour disorder is strongly associated with
Parkinson's disease, and is characterised by decreased muscle activity during REM
sleep.
E. Patients with Restless Legs Syndrome have worsening symptoms by night that cannot
be improved by moving around.
Question 275
Which of the following conditions is NOT associated with a higher incidence in smokers?
A.
B.
C.
D.
E.
Atherosclerosis.
Rheumatoid arthritis.
Age-related macular degeneration.
Oral cancer.
Cystic fibrosis.
Question 276
Gas exchange in the lungs happens by which of the following process:
A.
B.
C.
D.
E.
Osmosis
Diffusion
Exocytosis
Active Transport
Ion channels
Question 277
Under normal conditions, the rate of sweat secretion from a sweat gland is driven by
calcium in the secretory coil. Which family of chloride channels will be activated through
this process?
A. The volume-activated chloride channels.
B.
C.
D.
E.
The ClC family.
The TMEM16a family.
The CFTR family.
None of the above.
Question 278
Which of these groups of symptoms are NOT consistent with prolonged wakefulness?
A.
B.
C.
D.
E.
Fall in body temperature, blurred vision and mood changes.
Fall in body temperature, slurred speech and ataxia.
Fall in body temperature, mood changes and visual hallucinations.
Mood changes, memory lapses and weight loss.
Mood changes, impaired concentration and irritability.
Question 279
Which of the following most adequately describes the physiological requirements for
exercise without breathlessness?
A.
B.
C.
D.
E.
Air must go in and out of the respiratory system.
Air and blood must meet.
Oxygen must be available to muscle.
Muscle must be strong enough to allow movement.
All of the above.
Question 280
Stress regarded as 'a reciprocal process between the environment and the person', is the
central tenant of which model of stress?
A.
B.
C.
D.
E.
The willingness model
The cognitive dynamic model
The response based model
The transaction model
The stimulus based model
Question 281
Barry, a 75yo retired carpenter, comes to your clinic. He tells you that he's started to find
he's having trouble hearing what his wife is saying while they watch TV, and sometimes his
daughter complains that he doesn't respond when she's talking to him. He tells you that
about 5 years ago, he went to the doctor after a loud noise from a piece of machinery
caused a ringing in his ear, and he was told he had a small hole in his eardrum - the hole
was about 20% of the total surface. He is very worried that this is the cause of his current
hearing loss. Which of the following is the most likely cause for his hearing loss?
A.
B.
C.
D.
The previous trauma to his eardrum.
A subsequent trauma to his eardrum.
Age-related degradation of the inner row of hair cells of his inner ear.
Age-related degradation of the outer row of hair cells of his inner ear.
E. None of the above.
Question 282
Childhood trauma can be particularly difficult with children lacking the emotional
resilience of adults. Which of the following manifestations of trauma are more
characteristic in children that adults?
A.
B.
C.
D.
E.
Altered Arousal
Repetitive Behaviours
Negative Cognitions and Mood
Avoidance of Stimuli Associated with Trauma
Re-experiencing the Trauma
Question 283
Which of the following is TRUE with regards to ?-agonists?
A.
B.
C.
D.
E.
They cause relaxation of airway smooth muscle leading to bronchodilation.
They inhibit the release of histamine and leukotrienes.
They increase the beat frequency of cilia within the upper airways.
They cause a reduction in the levels of acetylcholine acting on the smooth muscle cells.
All of the above.
Question 284
Which muscle needs to relax during swallowing to allow food to go down the oesophagus?
A.
B.
C.
D.
E.
Palatopharyngeus
Salpingopharyngeus
Thyropharyngeus
Cricopharyngeus
Stylopharyngeus
Question 285
A patient's blood gases reveal the following:pH 7.39Partial pressure of oxygen: 44
mmHgPartial pressure of carbon dioxide: 67 mmHgBase Excess +10 mmol.How would you
classify their acid/base status?
A.
B.
C.
D.
E.
Their pH is normal so they are neither acidotic or alkalotic.
A primary respiratory alkalosis which is compensated.
A primary respiratory acidosis which is compensated.
A primary metabolic alkalosis which is compensated.
A primary metabolic acidosis which is compensated.
Question 286
Patients with Cystic Fibrosis are affected by abnormal transport of chloride across an
epithelium due to a mutation on the CFTR gene. In sweat glands, this means that chloride is
not reabsorbed, and thus they produce salty sweat. Under experimental conditions, betaagonists can be used to promote secretions to discover a defect in the CFTR gene. What
effect of ?-agonists activates CFTR chloride channels and consequently enables
investigators to observe a defect?
A. β-agonists increases concentration of calcium to activate CFTR chloride channels.
B. β-agonists cause Protein Kinase A activation and hence cyclic AMP (cAMP) production,
which activates CFTR chloride channels.
C. β-agonists increase the flow rate of sweat secretion and therefore promote CFTR
chloride channel activation.
D. β-agonists promote osmosis by increasing the leakage of CFTR chloride channels.
E. None of the above.
Question 287
Which of the following is not a measure of effect?
A.
B.
C.
D.
E.
relative risk
relative risk reduction
odds ratio
absolute risk reduction
hazard risk
Question 288
Which of the following regarding exocrine glands is correct?
A. Examples of discrete exocrine glands include sweat glands, salivary glands and the
kidneys.
B. They are derived from epithelial cells.
C. They characteristically have sodium-potassium ATPase transporters on their apical
surface.
D. Deficient exocrine function is a leading cause of interstitial lung disease.
E. Cystic fibrosis is due to a congenital absence of exocrine glands.
Question 289
Mast cells are vital in the acute response to antigen in asthma.Which of the following
concerning mast cells in asthma and subsequent mast cell responses is correct?
A. Leukotrienes are pre-formed mediators released from mast cells during degranulation
B. Sodium cromoglycate is a suicide inhibitor of mast cells, inducing mast cell apoptosis
C. Cross-linking of IgEs on the surface of mast cells by antigen stabilises the cells and
inhibits degranulation events
D. A mannitol challenge causes a reduction in mucosal cell osmolarity, eventually leading
to mast cell-mediated bronchial smooth muscle contraction
E. Nedocromil has a direct effect on mast cells to reduce the % fall in FEV1 in response to
an antigen challenge in the bronchial tree
Question 290
In regards to the effect of light in the regulation of sleep and wakefulness, which statement
is correct
A.
B.
C.
D.
E.
Light increases melatonin secretion
Light shifts the timing of the circadian system
Has a slow, gradual effect on alertness and performance
Blind people cannot synchronise to the light/dark cycle
Light stimulates the suprachiasmatic nucleus (SCN) in the amygdala
Question 291
Which of the following is incorrect?
A. Central chemoreceptors respond to changes in blood gas levels slower than peripheral
chemoreceptors.
B. Peripheral chemoreceptors are both in the carotid bodies and aortic bodies.
C. Central chemoreceptors are located in the medulla.
D. Peripheral chemoreceptors detect changes in both arterial PO2 and PCO2.
E. Central chemoreceptors can detect changes in PO2, PCO2 and pH.
Question 292
Which of the following statements about primary and secondary surveys is incorrect?
A. Ensuring a patent airway is of utmost concern in emergency medicine.
B. A thorough head-to-toe examination should be performed in all secondary surveys to
identify all injuries.
C. Primary survey includes initial trauma radiological examinations – lateral c-spine, chest
and pelvic x-rays and a bedside ultrasound examination where the mechanism, signs or
symptoms suggest internal injuries.
D. The secondary survey is a review of the patient and all investigation results which is
undertaken approximately 24 hours after the initial trauma.
E. Testing for neurological disability should initially be limited to GCS, pupillary response
and limb power, sensation and reflexes
Question 293
Oscar is a 13 year old boy who presents to you, his local GP with complaints of more
frequent attacks of dyspnoea and wheezing related to his asthma caused by house dust
mite exposure. Currently, the only medication he uses to treat his asthmatic symptoms is
Ventolin (salbutamol). Due to the increased frequency of Oscar's attacks, you consider
prescribing inhaled corticosteroids (ICS). Which of the following regarding ICS treatment is
incorrect?
A. ICS treatment may act to reduce the circulating numbers of eosinophils and mast cells
in asthmatic patients
B. Fixed dose combinations of ICS and long acting β-agonists are available for asthma
treatment
C. ICS doses may be tapered off once the asthamtic patient reports improvement in
nocturnal symptoms
D. A patient on ICS treatment will require exposure to more triggering antigen to produce
an equal % fall in FEV1 compared to if the patient was not on ICS treatment
E. Budesonide, fluticasone and beclomethasone are all approved ICSs for the treatment of
asthma
Question 294
Which of the following is one of the margins of the thoracic inlet?
A.
B.
C.
D.
E.
The clavicles anteriorly
The first pair of ribs laterally
The apices of the lungs
The sternocleidomastoid muscle laterally
The C7 vertebra posteriorly
Question 295
If someone watches TV late at night for three nights in a row, what effect will this have on
their sleep pattern on the fourth night?
A. Their sleep drive will be reduced, therefore they will go to bed later and wake up later.
B. Light into the suprachiasmatic nucleus (SCN) will maintain the release of melatonin,
meaning they will feel sleepier later.
C. Light into the suprachiasmatic nucleus (SCN) will inhibit the release of melatonin,
meaning they will feel sleepier later.
D. Their homeostatic drive to sleep will build up, meaning they will be tired earlier.
E. None of the above.
Question 296
Claire gave birth to her first child, Robert, 2 years ago. Robert was born with cystic fibrosis.
Claire has brought him to your clinic because he has symptoms consistent with a lung
infection. Which of the following organisms is most likely the causative agent?
A.
B.
C.
D.
E.
Pseudomonas aeruginosa
Staphylococcus pneumoniae
Haemophilus influenzae
Candida albicans
Staphylococcus aureus
Question 297
Which of the following is NOT an important factor when writing a medical certificate for a
patient?
A.
B.
C.
D.
E.
A judgement of the patient's actions.
Observation of the patient's privacy.
Treatment of the cause of the presenting complaint.
Education of the patient regarding how to prevent recurrence.
Maintaining integrity as a medical practitioner.
Question 298
As a first year intern, you accidentally administer pure oxygen for a prolonged period of
time to a patient with otherwise normal lung function. What biochemical process would
you expect NOT to occur within this patient?
A. Oxidation of thioredoxin leading to dissociation of apoptosis signal-related kinase 1.
B. Enhanced expression of pyruvate dehydrogenase kinase restricting entry of pyruvate
into the citric acid cycle.
C. Lower rates of glutathione conversion to an oxidised form.
D. Increased serum pH.
E. The activation of antioxidant response enzymes.
Question 299
A patient's blood gases reveal the following:pH: 7.55Partial pressure of oxygen: 117
mmHgPartial pressure of carbon dioxide: 21 mmHgBase Excess: -4 mmol.They are
breathing room air in the Emergency Department of RPA Hospital, Sydney (assume sea
level). What is the alveolar-arterial (A-a) gradient for this patient?
A.
B.
C.
D.
E.
-21.75
-6.75
6.75
21.75
30.75
Question 300
Many genetic and environmental factors have been found to predispose an individual
towards the development of asthma. Which of the following HAVE NOT been found to be
positively associated with subsequent asthma development?
A.
B.
C.
D.
E.
Early childhood infections, especially viral.
Maternal smoking during pregnancy
High omega-6 intake, and low omega-3 intake
Antibiotic use under the age of 2
Environmental pollutants such as sulphur dioxide.
Question 301
John, a 24-year-old medical student, comes into your clinic complaining of reduced hearing.
You perform Rinne and Weber testing using a tuning fork. The results are as follows:Rinne
left ear: air conductance better than bone conductanceRinne right ear: air conductance
worse than bone conductanceWeber: sound is heard more strongly in the right earWhat do
you think is the most likely mechanism behind his hearing impairment?
A.
B.
C.
D.
E.
Sensorineural hearing impairment in right ear.
Sensorineural hearing impairment in left ear.
Conductive hearing loss in right ear.
Conductive hearing loss in left ear.
None of the above.
Question 302
You are seeing an Indigenous patient in a rural clinic in Alice Springs who has come to you
for the first time. On what knowledge base are you going to rely most to tailor your clinical
understanding and to ensure that you give this patient the most effective treatment?
A. A randomised control study that was conducted with a sample base purely from the
North Shore of Sydney.
B. An isolated case involving the same disease that you remember a colleague telling you
about.
C. A newspaper report on the condition of health care in Alice Springs.
D. A detailed history of the patient's diet, living conditions, social history and cultural
preferences
E. Cultural training that you have completed for a specific Indigenous nation, one from
which you are unsure your patient originates.
Question 303
Elisa is a 14-year-old girl with cystic fibrosis. You have recently diagnosed her with a lung
infection and identified the causative organism as methicillin-sensitive Staphylococcus
aureus (MSSA). Which of the following specifically relating to MSSA infections is important
to consider before beginning treatment?
A.
B.
C.
D.
MSSA is resistant to antibiotics and can only be treated with anti-staphylococcal agents.
Antibiotic treatment may increase the risk of antibiotic resistance developing.
MSSA is an intracellular pathogen, and can therefore evade the actions of antibiotics.
14 year old girls do not get infected with MSSA. There was likely a problem with the
investigation which identified MSSA as the causative organism.
E. Treatment with anti-staphylococcal agents may predispose her to colonisation with P.
aeruginosa.
Question 304
Which of the following is NOT an element of open disclosure following an adverse event?
A.
B.
C.
D.
E.
An apology and expression of regret.
A factual explanation of what happened.
An opportunity for the patient do discus their thoughts.
An explanation that the cause of the event is unknown and shouldn't happen again.
An explanation of steps taken to prevent a further adverse event.
Question 305
Which of the following anatomical landmarks is not at the level of the xiphisternal joint?
A.
B.
C.
D.
E.
The inferior limit of the thoracic cavity.
The central tendon of diaphragm.
The inferior limit of the liver.
The inferior border of the heart.
The superior limit of the liver.
Question 306
Homeostatic drive increases as we're awake, circadian drive compliments the homeostatic
drive in the beginning so as to maintain wakefulness. Once the homeostatic drive reaches a
critical point later in the evening, we become ready for sleep.According to the statement
above, which of the following best characterizes the relationship between homeostatic
drive and circadian drive before the homeostatic drive reaches the critical point?
A. Circadian drive tends to wane in the evening while the homeostatic drive keeps
accumulating.
B. Circadian drive tends to increase in the evening while the homeostatic drive keeps
accumulating.
C. Circadian drive tends to wane as the homeostatic drive wanes in the evening.
D. Circadian drive tends to increase in the evening while the homeostatic drive wanes.
E. Both the circadian drive and the homeostatic drive wane as the day passes.
Question 307
Why might a baby be admitted to hospital for RSV infection, whereas an adult would not?
A.
B.
C.
D.
Babies are more likely to be exposed to RSV from other children.
Adults are immunized against RSV and generally do not become infected.
Babies suffer more severe diarrhea from RSV than adults.
The standard adult treatment for RSV is antibiotics which have side effects too strong
for babies.
E. Babies has smaller airway structure and weaker respiratory muscles that make
breathing with RSV difficult.
Question 308
Management of obstructive sleep apnoea can involve the following except:
A.
B.
C.
D.
E.
Nasal continuous positive airway pressure.
Oral temazepam.
Oral prosthesis.
Weight reduction.
Avoidance of alcohol.
Question 309
Which of the following describes a lung affected with Emphysema best?
A.
B.
C.
D.
E.
The lungs are more elastic, more compliant and lung volumes are greater.
The lungs are less elastic, more compliant and lung volumes are smaller.
The lungs are less elastic, more compliant and lung volumes are greater.
The lungs are more elastic, less compliant and lung volumes are greater.
None of the above.
Question 310
On the fourth day heel prick tests are conducted on neonates. Which of the following is not
true about the heel prick test?
A. It measures the Na/Cl ions for CF
B.
C.
D.
E.
It tests for hypothyroidism
it tests for phenyalanine
it tests for galactasemia
it also tests for rare metabolic conditions
Question 311
Which of the following is true with respect to the blood supply of the thorax?
A.
B.
C.
D.
E.
The bronchial arteries arise from the pulmonary arteries.
The internal thoracic arteries branch off the arch of the aorta.
The intercostal arteries run superior to their corresponding veins.
The accessory hemiazygos vein run along the left side of the vetebral column.
The azygos vein drains into the inferior vena cava.
Question 312
Which of the following is NOT a possible clinical sign of a tension pneumothorax?
A.
B.
C.
D.
E.
A deviated trachea.
Hyperresonance on percussion.
Hypoexpanded chest that moves dynamically with respiration.
Decreased breath sounds.
Decreased tactile fremitus.
Question 313
What is not a potential result of a tension pneumothorax?
A.
B.
C.
D.
E.
CNS hypoxia
CV collapse
Clubbing
Tracheal displacement
Mediastinal compression
Question 314
Your patient is suffering from dysphagia following a stroke.Which cranial nerve(s) are
responsible for the gag reflex?
A. The vagus nerve (CN X).
B. Sensory fibres of the mandibular nerve (CN V3) and motor fibres of the vagus nerve (CN
X).
C. Sensory fibres from the glossopharyngeal nerve (CN IX) and motor fibres from the
vagus nerve (CN X)
D. Sensory fibres from the vagus nerve (CN X) and motor fibres from the glossopharyngeal
nerve (CN IX)
E. Sensory fibres from the mandibular nerve (CN V3) and motor fibres from the
glossopharyngeal nerve (CN IX)
Question 315
Which of the following does NOT make up the contours of the mediastinum on a frontal
chest X-ray?
A.
B.
C.
D.
E.
Aortic knuckle
Right ventricle
Left ventricle
Superior vena cava
Left atrial appendage
Question 316
Mandibular advancement splints assist breathing in obstructive sleep apnoea by:
A.
B.
C.
D.
E.
Opening the mouth to let more air in.
Pushing down the tongue to prevent the patient from swallowing it.
Gently widening the airway over a long time, like the actions of braces.
Pushing the jaw forwards to pull open the pharynx antero-posteriorally.
Pumping air into the lungs to keep the larynx and small airways open upon expiration.
Question 317
On a T2 weighted median plane MRI of the head and neck, the greatest signal intensity will
be produced by
A.
B.
C.
D.
E.
Cerebrospinal fluid
Grey matter of the nervous system
White matter of the nervous system
Adipose tissue
Cranial sinuses
Question 318
Central sleep apnoea is accompanied by
A.
B.
C.
D.
E.
an absence of diaphragm EMG and thoraco-abdominal wall movement
continued or raised diaphragm EMG and thoraco-abdominal wall movement
continued or raised diaphragm EMG, but not thoraco-abdominal wall movement
paradoxical thoraco-abdominal wall movement
none of the above
Question 319
Which of the following is CORRECT regarding cessation of smoking and its effects on life
expectancy?
A.
B.
C.
D.
E.
Cessation of smoking at 20 years old adds approximately 90 years to life expectancy.
Cessation of smoking at 30 years old adds approximately 20 years to life expectancy.
Cessation of smoking at 40 years old adds approximately 15 years to life expectancy.
Cessation of smoking at 50 years old adds approximately 5-6 years to life expectancy.
Cessation of smoking at 60 years old adds approximately 1-2 years to life expectancy.
Question 320
Nicotine is the primary addictive drug in cigarettes. Which of the following is NOT a direct
effect of nicotine on the body?
A.
B.
C.
D.
E.
Increased sympathetic activity.
Increased heart rate and blood pressure.
Peripheral vasoconstriction.
Binds to haemoglobin causing decreased oxygen content in blood.
Can result in premature births and low birth weight.
Question 321
Kerley B lines are a sign of which pathology?
A.
B.
C.
D.
E.
Pulmonary embolism
Pneumonia
Pulmonary oedema
Pulmonary fibrosis
Pleuritis
Question 322
'Kerley B lines' are a sign that most likely indicate which respiratory pathology?
A.
B.
C.
D.
E.
Pulmonary embolism.
Pulmonary oedema.
Pneumonia.
Pulmonary fibrosis.
Emphysema.
Question 323
The respiratory quotient describes the relationship between the amount of CO2 produced
by a process and the amount of O2 consumed. For the metabolism of sugars and fats, the
respiratory quotient is approximately:
A.
B.
C.
D.
E.
0.8
1
0.5
0.1
0.5
Question 324
Which of the following is the LEAST significant contributor to the shorter life expectancy in
Indigenous people (as compared to non-Indigenous people) living in the Northern
Territory?
A.
B.
C.
D.
E.
Cardiovascular disease
Injury
Chronic respiratory conditions
Diabetes mellitus
Melanoma
Question 325
Which of following lung volumes capacities can not be measured directly using spirometry?
A.
B.
C.
D.
E.
Expiratory reserve volume
Functional residual volume
Inspiratory reserve volume
Tidal volume
Vital capacity
Question 326
Which of the following has contributed to the rise of the smoking epidemic at the turn of
the 20th century?
A.
B.
C.
D.
E.
Introduction of the Bosnack cigarette rolling machine.
Invention of the safety match.
Mass marketing campaigns.
A, B and C.
A and C only.
Question 327
Which of the following is NOT a symptom commonly associated with Narcolepsy?
A.
B.
C.
D.
E.
Excessive daytime sleepiness
Periodic movements in sleep
Sleep paralysis
Cataplexy
Sleep hallucinations
Question 328
What makes up Vital Capacity?
A.
B.
C.
D.
The total lung capacity (TLC) minus the expiratory reserve volume (ERV)
The total lung capacity (TLC) minus the functional residual capacity (FRC).
The inspiratory capacity (IC) plus the functional residual capacity (FRC).
The inspiratory capacity (IC) plus the expiratory reserve volume (ERV) plus the tidal
volume (Vt)
E. The inspiratory capacity (IC) plus the expiratory reserve volume (ERV).
Question 329
In a patient with muscular dystrophy, what happens to their residual volume and diffusing
capacity?
A.
B.
C.
D.
E.
Residual volume and diffusing capacity remains the same
Residual volume increases and diffusing capacity decreases
Residual volume increases and diffusing capacity remains the same
Residual volume decreases and diffusing capacity decreases
Residual volume decreases and diffusing capacity remains the same
Question 330
With regards to obstructive airway disease, which of the following is FALSE?Note that
FEV1 stands for forced expiratory volume in one second, and FVC stands for forced vital
capacity.
A. Patients with emphysema suffer a grossly reduced FEV1 compared to normal subjects.
B. The ratio of FEV1 to FVC in patients with obstructive airway disease is decreased below
80%.
C. Use of a bronchodilator in patients suffering from airways obstruction due to asthma
will return their FEV1 and FVC values back to normal.
D. Use of a peak flow meter by patients suffering from asthma is a useful way of
monitoring airways obstruction and response to asthma treatment at home.
E. In emphysema, expiratory flow rates are reduced due to loss of pulmonary elastic
recoil.
Question 331
Which of the following is not a Bradford Hill absolute criterion for causality?
A.
B.
C.
D.
E.
The exposure precedes the outcome.
The outcome precedes the exposure.
The association is unlikely to be due to chance.
The association is unlikely to be due to bias.
The association is unlikely to be due to confounding variable.
Question 332
A patient is brought into the emergency department, following a motor vehicle accident
(MVA), which of the following is of the highest priority?
A.
B.
C.
D.
E.
Assess gross motor control to check for neurological damage.
Palpate the trachea for deviation to assess for tension pneumothorax.
Perform an X-ray of chest, head and abdomen to check for occult bleeds.
Assess blood pressure to monitor shock and blood loss.
Strip the patient and perform a log roll to ensure no wounds have been missed.
Question 333
The calculation known as attributable risk provides an estimate of what quantity?
A. The incidence of a disease that is due to a certain exposure.
B. The ratio of the probability of developing a disease among those exposed to a risk
factor, compared with the probability of developing the outcome if the risk factor is not
present.
C. The odds of the experimental group showing positive (or negative) effects of an
intervention or exposure, in comparison to the control group.
D. The risk of developing some new condition within a specified period of time.
E. All of the above.
Question 334
Which of the following mediators is NOT found in type I hypersensitivity?
A.
B.
C.
D.
E.
Prostaglandin D2
Histamine
Leukotriene B4
TNFa
Leukotriene E4